You are on page 1of 148

Basic Concepts

1. The Central Government has been empowered by entry of the Union list of Schedule VII of
the constitution of India to levy tax on income other than agricultural income.
(A) 84
(B) 81
(C) 82
(D) 84

C;

Solution: Entry 82 of the Union List i.e., List I in the Seventh Schedule to Article 246 of the
Constitution of India has given the power to the Parliament to make laws on taxes on
income other than agricultural income.

2. Describe the status of the following person (i.e. individual, HUF, Firm, Company etc.) X and Y
are legal heirs of Z. Z died in 2021 and X and Y carry on his business without entering into a
partnership.
(A) Firm
(B) Limited Liability Partnership
(C) Company
(D) Body of Individual

D;

Solution: Body of individual it denotes the status of persons like executors or trustees who
merely receive the income jointly and who may be assessable in like manner and to the
same extent as the beneficiaries individually.
3. XYZ LLP falls under category of person -
(A) Individual
(B) Partnership firm
(C) Company
(D) Association of person

B;

Solution: The terms 'firm', 'partner' and 'partnership' have the same meanings as assigned
to them in the Indian Partnership Act, 1932. In addition, the definitions also include the
terms limited liability partnership, a partner of limited liability partnership as they have
been defined in the Limited Liability Partnership Act, 2008.

4. A person follows calendar year for accounting. For taxation, he has to follow:
(A) Calendar year only - 1st January to 31st December
(B) Financial year only - 1st April to 31st March
(C) Any of the Calendar or Financial year as per his choice
(D) He will to follow extended year from 1st January to next 31st March (a period of 15
months)

B;

Solution: For the purpose of taxation uniform financial year from only - 1st April to 31st
March is to be followed.

5. In case of non-residents engaged in shipping business in India income earned during the
financial year is —
(A) Taxable in India the same financial year
(B) Taxable in India the relevant assessment year
(C) Not taxable in India in the same financial year
(D) Not taxable in India.

A;

Solution: In case of non-residents engaged in shipping business in India income earned


during the financial year is taxable in India the same financial year.

6. Which amongst the following is not a head of Income?


(A) Salaries
(B) Income from house Property
(C) Capital gains
(D) Income from exports

D;

Solution: According to Section 14 of the Act, all the incomes are categorised in the following
five heads —

(i) Salaries [Section 15-17];

(ii) Income from House Property [Section 22-27];

(iii) Profits and Gains of Business or Profession [Section 28-44DB];

(iv) Capital Gains [Section 45-55A]; and

(v) Income from Other Sources [Section 56-59]

Hence, income from exports is not a head of income.


7. Mr. X is resident Individual (30 years of age) having total Income of ₹ 2,01,00,000, the tax
payable by him is —
(A) ₹ 67,84,375
(B) ₹ 75,95,250
(C) ₹ 70,55,750
(D) ₹ 69,51,750

C;

Solution: Tax on total income of ₹ 2,01,00,000 (including surcharge @ 25%) = 73,03,125

Tax on ₹ 2,00,00,000 (including surcharge @ 15%) = 66,84,375

Marginal Relief = ₹ 73,03,125 - (₹ 66,84,375 + 1,00,000) = ₹ 5,18,750

Tax liability = (₹ 73,03,125 —₹ 5,18,750) + 4% of ₹ 67,84,375 =₹ 70,55,750

8. The income-tax payable by a Mrs. Swati Non-Resident Individual (aged 65 years) for A.Y.
2021-22 if her total income is ₹ 2,75,000 will be :
(A) Nil
(B) ₹ 1250
(C) ₹ 1,300
(D) ₹ 2,600

C;

Solution: The tax liability will be —

On first ₹ 2,50,000 = Nil

On next ₹ 25,000 = ₹ 1,250

Total tax = ₹ 1,250


Add: HEC@ 4% = ₹ 50

Tax including HEC @ 4% = ₹ 1,300. [No rebate under Section 87A is allowed to Non-
resident]

9. Long term capital Gains on sale of equity shares listed in recognized stock exchange on
which STT is payable at the time of purchase and sale of shares is chargeable to tax @
in excess of Rs under section 112A.
(A) 10%, Rs 1,00,000
(B) 15%, Rs 1,00,000
(C) 20%, Rs 10,00,000
(D) 30% Rs 10,00,000

A;

Solution: Long term capital Gains on sale of equity shares listed in recognized stock
exchange on which STT is payable at the time of purchase and sale of shares is chargeable
to tax @ 10% in excess of Rs 1,00,000 under section 112A.

10. If a resident Individual (40 years of age) has total Income is ₹ 5,00,50,000, the marginal
relief available to the him is —
(A) ₹ Nil.
(B) ₹ 17,98,050
(C) ₹ 29,450
(D) ₹ 17,48,050

D;

Solution: Tax on total income of ₹ 5,00,50,000 (including surcharge @ 37%) = ₹ 2,03,13,675


Tax on ₹ 5,00,00,000 (including surcharge @ 25%) = ₹ 1,85,15,625

Marginal Relief = ₹ 2,03,13,675 - (₹ 1,85,15,625 + ₹ 50,000) = ₹ 17,48,050

Tax liability = (₹ 2,03,13,675 — ₹ 17,48,050) + 4% of ₹ 1,85,65,625= ₹ 1,93,08,250


Assesment of Trust and Companies

1. The advancement of any other object of general public utility shall be regarded as charitable
purpose even if it involves carrying on of trade/commerce/business for a cess or fee or any
other consideration, if the gross receipts thereof does not exceeds of total
receipts of such charitable institution.
(A) 15%
(B) 20%
(C) ₹ 20 lakhs
(D) ₹ 25 lakhs

2. A public charitable trust registered under section 12A of Income-tax Act, for the previous
year ending 31-3-2021, derived gross income of ₹ 16 lakhs, which consist of the following:

Particulars ₹

(i) Income from properties held by trust (Net) 5,00,000

(ii) Income (New) from business (incidental to main objects) 4,00,000

(iii) Voluntary contribution from public 7,00,000

The trust applied a sum of ₹ 11.60 lakhs towards charitable purposes during the year.
Determine the taxable income of the trust of the assessment year 2021-22.
(A) ₹ 2,00,000
(B) Nil
(C) ₹ 4,40,000
(D) ₹ 6,00,000
3. Ramji Charitable trust had sold a capital asset costing Rs. 70,000 on 13 th June, 2020 for Rs.
1,50,000. It purchased new asset on 1st July 2020 for Rs. 1,20,000. The amount taxable as
capital gains for Ramji Charitable Trust in A.Y. 2021-22 is -
(A) ₹ 80,000
(B) Nil, because of charitable trust
(C) ₹ 30,000
(D) ₹ 40,000

4. A charitable acquired two air-conditioners for ₹ 1,40,000 on 10th June, 2020. It claimed the
acquisition as application of income. The amount it can claim by way of depreciation for the
said air-conditioners for the AY 2021-22 is –
(A) ₹ 21,000
(B) ₹ 1,40,000
(C) ₹ 35,000
(D) Nil

5. In case of any registered trust or institution for which registration is effective in the previous
year, they cannot claim any exemption under any provision of section 10 [other than that
relating to exemption of ].
(A) Section 10(1) and 10(23C)
(B) Section 10(34)
(C) Section 10(35)
(D) Section 10(15)
6. Under which of the following circumstances levy of tax on accreted income is attracted in
respect of trust or institution registered under Section 12AA.
(A) conversion of the trust or institution into a form not eligible for grant of registration
under section 12AA.
(B) Merger with an entity not having similar objects and registered under section 12AA.
(C) Non-distribution of assets on dissolution to any charitable institution registered under
section 12AA or approved under section 10(23C) within a period of 12 months from the
end of the month in which the dissolution takes place.
(D) All of the above.

7. Compute the tax liability for A.Y. 2021-22 of XYZ. Inc. a foreign company having total income
amounting ₹ 10,01,00,000 for the year ending 31-03-2021.
(A) ₹ 4,37,23,680
(B) ₹ 4,25,36,000
(C) ₹ 2,91,49,120
(D) ₹ 3,34,17,380

8. XYZ Ltd., a domestic company, purchases its own unlisted shares on 4 th July, 2020. The
consideration for buyback amounted to ₹ 21 lakh, which was paid on the same day. The
amount received by the company two years back for issue of such shares determined in the
manner specified in Rule 4OBB was ₹ 13 lakh. Compute the interest, if any, payable if
additional income tax is paid to the credit of the Central Government on 29 th September,
2020.
(A) ₹ 4,471
(B) ₹ 5,591
(C) ₹ 8,383
(D) ₹ 6,706
9. The tonnage income of qualifying ship having net tonnage 25000 tons which operated for
150 days is .
(A) ₹ 26,25,000
(B) ₹ 17,35,500
(C) ₹ 19,87,500
(D) ₹ 10,87,500

10. A registered charitable trust has applied ₹ 50 lakh and claimed it as application of income.
This application includes —
(i) cash payment of ₹ 40,000,
(ii) cash donation amounting ₹ 50,000 as donation to other charitable institution and
(iii) consultancy fee of ₹ 2,50,000 by an account-payee cheque to Z (a yoga instructor).
One this consultancy fee, tax is not deducted at source during the Financial Year
2020-21.

The amount which will be regarded as application for the purpose of claiming exemption
under Section 11 will be —

(A) ₹ 48,35,000
(B) ₹ 50,00,000
(C) ₹ 46,60,000
(D) ₹ 49,10,000
Answers

1. B;

Solution: The advancement of any other object of general public utility shall be regarded as
charitable purpose even if it involves carrying on of trade/ commerce/ business for a cess or
fee or any other consideration, if the gross receipts thereof does not exceed 20% of total
receipts of such charitable institution.

2. A;

Solution: Computation of taxable income of the trust —


Particulars

Income from properties held by trust (net) 5,00,000

Income (computed) from business (incidental to main objects) 4,00,000

Voluntary contributions from public 7,00,000

Income from property held under trust 16,00,000

Less: 15% set apart under section 11(1)(a) for future application 2,40,000

Balance Income 13,60,000

Less: Income applied by the trust towards charitable purposes. 11,60,000

Taxable income of the trust 2,00,000

3. C;

Solution: Computation of taxable capital gains—


Particulars ₹

Full value of consideration 1,50,000

Cost of acquisition of asset transferred 70,000

Capital gains 80,000

Less: Exemption u/s 11(1A) to the extent of the lower of the following i.e. (a) 50,000
Entire capital gains (₹ 80,000); or (b) [Cost of new capital asset i.e. ₹ 1,20,000
Less Cost of asset transferred i.e. ₹ 70,000] = ₹ 50,000

Taxable Capital Gains 30,000

4. D;

Solution: Where any income is required to be applied or accumulated, then, for such
purpose the income shall be determined without any deduction or allowance by way of
depreciation or otherwise in respect of any asset, acquisition of which has been claimed as
an application of income under this clause in the same or any other previous year. Hence,
admissible depreciation shall be nil.

5. A;

Solution: In case of any registered trust or institution for which registration is effective in
the previous year, they cannot claim any exemption under any provision of section 10
[other than that relating to exemption of Section 10(1) and 10(23C).

6. D;
Solution: Where in any previous year, a trust or institution registered u/s 12AA has—

(a) converted into any form which is not eligible for grant of registration under section
12AA;
(b) merged with any entity other than an entity which is a trust or institution having objects
similar to it and registered under section 12AA; or
(c) failed to transfer upon dissolution all its assets to any other trust or institution
registered under section 12AA or to any fund or institution or trust or any university or
other educational institution or any hospital or other medical institution referred to in
Section 10(23C)( iv)/(v)/(vi)/(via), within a period of 12 months from the end of the
month in which the dissolution takes place, tax on accreted income is attracted.

7. B;

Solution: Computation of total tax liability –

Particulars ₹

Assessee XYZ Inc.

Total Income 10,01,00,000

Tax 4,00,40,000

Surcharge 20,02,000

Tax Incl surcharge 4,20,42,000

Marginal relief 11,42,000

Balance tax 4,09,00,000

HEC @ 4% 16,36,000
Total tax (rounded off) 4,25,36,000

8. B;

Solution: Computation of interest —

Particulars ₹

Total Consideration paid on Buy back of shares 21,00,000

Less: Amount received on issue of shares 13,00,000

Amount liable for tax 8,00,000

Tax payable under Section 115QA @ 23.296% (rounded off) 1,86,370

Due date of payment of tax [4th July 2020 + 14 days] 18-07-2020

Actual date of deposit of tax 29-09-2020

Delay in months including part of months 3 months

Interest @ 1% p.m. [ 1,86,370 x 1% x 3] 5,591

9. B;

Solution: The daily tonnage income of a qualifying ship shall be computed on the basis of
the following table —

Qualifying ship having net tonnage Amount of daily tonnage income

Up to 1,000 ₹ 70 for each 100 tons


Exceeding 1,000 but not more than 10,000 ₹ 700 plus ₹ 53 for each 100 tons exceeding
1,000 tons

exceeding 1,000 but not more than 25,000 ₹ 5,470 plus ₹ 42 for each 100 tons
exceeding 10,000 tons

Exceeding 25,000 ₹ 11,770 plus ₹ 29 for each 100 tons


exceeding 25,000 tons

Hence, the income shall be computed as under [₹ 5,470 + ₹ 15,000 x 42/100] x 150 days = ₹
17,65,500.

10. A;

Solution: For the purposes of determining the amount of application under Section
11(1)(a)/(b), the provisions of Section 40(a)(ia) and Section 40A(3)/(3A), shall, mutatis
mutandis, apply as they apply in computing the income chargeable under the head "Profits
and gains of business or profession".

(i) Donation to another charitable trust amounting 50,000 in cash shall not be treated
as application of Income since it exceeds Z 10,000.
(ii) Similarly, cash payment of Z 40,000 and Z 75,000 (30% of Z 2.50,000) being
consultancy fees paid to Yoga instructor on which tax has not been deducted at
source shall not be treated as application of income for F.Y. 2020-21. Z 75,000 shall
be treated as application of income in F.Y. 2021-22 i.e. the year in which tax
deducted has been deposited.

Thus, the amount to be treated as application of Income —


Particulars ₹

Total application 50,00,000

Less: Donation to another charitable trust 50,000

Less: Cash payment 40,000

Less: Consultancy fees 75,000

Amount to be treated as application of income 48,35,000


Assessment of various entities exempt income

1. Mr. Shiva gifted a let-out building which fetches rental income of ₹ 10,500 per month to his
son's wife on 1-11-2020. The municipal tax of ₹ 6,000 on the property was paid on 10-1-
2021. The total income from all other sources (computed) amounts to ₹ 2,60,000 except
income from above said property. His total income chargeable to tax is :
(A) ₹ 3,11,450
(B) ₹ 3,44,000
(C) ₹ 3,80,000
(D) ₹ 3,33,500

2. Which of the following statement is true with reference to the provisions of the Income Tax
Act, 1961?
(A) Mr. A, a member of a HUF, received ₹ 10,000 as his share from the income of the HUF.
The same is to be included in his chargeable income.
(B) Pension received by a recipient of gallantry award, who was a former employee of
Central Government, is taxable under income-tax.
(C) Rent of ₹ 72,000 received for letting out agricultural land for a movie shooting is exempt
from tax.
(D) Any payment from an account opened in accordance with the Sukanya Samriddhi
Account Rules, 2014, made under the Government Savings Bank Act, 1873, shall not be
included in the total income of the assessee.

3. M/s PQR & Co., a firm carrying on business, furnishes the following particulars for the P.Y.
2020-21.
Particulars ₹

Book profits (before setting of unabsorbed depreciation and brought forward 2,70,000
business loss)

Unabsorbed depreciation of P.Y. 2014-15 1,20,000

Brought forward business loss of P.Y.2019-20 2,00,000

Compute the amount of remuneration allowable under section 40(b) from the book profit.

(A) ₹ 2,43,000
(B) ₹ 1,80,000
(C) ₹ 1,50,000
(D) Nil

4. Any interest, salary, bonus, commission due to or received by a partner from a partnership
firm is chargeable under the head—
(A) Profits and gains of business or profession
(B) Income from other sources
(C) Salaries
(D) None of the above.

5. A, B and C are the partners in a firm engaged in medical profession. For the year ended on
31st March, 2021, the book profits of the firm was calculated as ₹ 10,00,000. The maximum
amount admissible as remuneration to partners in accordance of provisions of Section 40(b)
is .
(A) ₹ 6,90,000
(B) ₹ 1,50,000
(C) Nil
(D) ₹ 2,70,000

6. An association of persons (AOP) has paid tax at the maximum marginal rate. Yash, a
member of AOP received ₹ 1 lakh to his share income. Such income is chargeable to tax in
his assessment @ —
(A) 10%
(B) Nil
(C) 30%
(D) 20%

7. Mr. Sankar received ₹ 50,000 as educational scholarship from Nehru Memorial Trust (a
charitable trust). The scholarship is to assist Mr. Sankar for pursuming M.A. (History) at
Jawaharlal Nehru University, New Delhi. The amount of scholarship liable to tax is :
(A) ₹ 50,000
(B) ₹ 10,000
(C) ₹ 25,000
(D) Nil

8. Dividend received by a real estate investment trust (REIT) from special purpose vehicle
(SPV) and distributed to its unit holders is—
(A) exempt in the hands of both the REIT and the unit holders unconditionally
(B) exempt in the hands of the REIT only if the SPV is a specified domestic company; exempt
In the hands of unit holders only if taxable in the hands of REIT
(C) exempt in the hands of the REIT only if the SPV is a specified domestic company; exempt
in the hands of unit holders only if exempt in the hands of REIT
(D) exempt in the hands of the REIT only if the SPV is a specified domestic company; exempt
unconditionally in the hands of unit holders

9. Y Co. Ltd. furnishes you the following information for the year ended 31-3-2021 :

Particulars ₹ in lakhs

Total turnover of Unit A located in Special Economic Zone which is in first 100
year of operation

Profit of the business of Unit A 30

Export turnover of Unit A 50

Total turnover of Unit B located in Domestic Tariff Area (DTA) 200

Profit of the business of Unit B 20

Compute deduction under section 10AA.


(A) ₹ 15,00,000
(B) ₹ 8,33,333
(C) ₹ 30,00,000
(D) ₹ 7,50,000

10. Income amounting to t 32,000 of a securitization trust from the activity of securitization in
hands of securitization trust will be .
(A) 32,000
(B) 20,000
(C) 3,200
(D) Exempt
Solutions

1. B;

Solution: As per Section 64(1)(vi), Income arising out of asset transferred without
adequate consideration to son's wife is liable to be clubbed in the hands of the assessee,
subject to the condition that the said relationship must exist at the time of both transfer
of property and accrual of income.

Computation of total income —

Particulars ₹ ₹

Income from house property : Gross Annual Value [₹10,500 x 12] 1,26,000

Less: Municipal Taxes paid 6,000

Net Annual Value 1,20,000

Less: Standard deduction u/s 24 [30% of NAV] 36,000 84,000

Income from all other sources 2,60,000

Gross Total Income/Total Income 3,44,000

2. D;

Solution: As per Section 10(11A), any payment from an account opened in accordance with
the Sukanya Samriddhi Account Rules, 2014, made under the Government Savings Bank Act,
1873, shall not be included in the total income of the assessee.
3. B;

Solution: Computation of book profits for the purpose of remuneration

Particulars ₹

Book profits (before setting of unabsorbed depreciation and brought forward 2,70,000
business loss)

Less: Unabsorbed depreciation of P.Y. 2014-15 70,000

Book Profits 2,00,000

Allowable remuneration 90% of ₹ 2,00,000 1,80,000

Note: From Book profits before setting of unabsorbed depreciation, brought forward
business loss shall be notionally set-off and from the balance profits, unabsorbed
depreciation shall be set-off to compute book profits.

4. A;

Solution: Any interest, salary, bonus, commission due to or received by a partner from a
partnership firm is chargeable under the head Profits and gains of business or profession.

5. A;

Solution: Maximum allowable remuneration as per Section 40(b) = 90% of ₹ 3,00,000 + 60%
of ₹ 7,00,000 = ₹ 6,90,000.

6. B;
Solution: When AOP is taxed at maximum marginal rate, share of the member in the
Income of AOP is exempt from tax.

7. D;

Solution: Education scholarship is exempt from tax

8. D;

Solution: Dividend received by a real estate investment trust (REIT) from special purpose
vehicle (SPV) and distributed to its unit holders is exempt in the hands of the REIT only If the
SPV Is a specified domestic company; exempt unconditionally in the hands of unit holders.

9. A;

Solution: Unit A located in Special Economic Zone is eligible for deduction under section
10AA of Act and such deduction is to be computed undertaking-wise. 100% of the profit
derived therefrom will be eligible as deduction assuming that the previous year falls within
the first 5 years commencing from the year of manufacture or production of articles or
things or provisions of services by the Unit in the SEZ.

Computation of the business income:

Particulars ₹

Profits of Unit A :

Profits earned 30,00,000


Less: Deduction u/s 10AA [WN] 15,00,000

Taxable Profits of Unit S 15,00,000

Profits of Unit B located in Domestic Tariff Area (i.e. non - SEZ area) 20,00,000

Income under the head 'Profits and Gains of Business or Profession' 35,00,000

Working Note: Deduction eligible u/s 10AA = 100% of Profits x Export turnover of the
Undertaking ÷ Total Turnover of the Undertaking = 100% of [30 x 50 ÷ 100] = ₹ 15 lakhs.

10. D;

Solution: Income of a securitization trust from the activity of securitization in hands of


securitization trust will be exempt from tax under Section 10(23DA).
Capital Gains

1. What are the conditions to be fulfilled for charging of income under the head capital gains :
(A) There must be a capital asset.
(B) There must be a transfer of such capital asset.
(C) The transfer of such capital asset has been affected during the previous year.
(D) All of the above.

D;

Solution: Section 45 provides that any profits or gains arising from the transfer of a capital
asset effected in the previous year will be chargeable to income-tax under the head 'Capital
Gains'. Such capital gains will be deemed to be the income of the previous year in which the
transfer took place.

2. In terms of section 2(42A), listed securities are treated as long-term capital asset, if they are
held for a period of more than—
(A) 12 Months
(B) 36 Months
(C) 24 Months
(D) 48 Months

A;

Solution: In terms of section 2(42A), listed securities are treated as long-term capital asset,
if they are held for a period of more than 12 months immediately preceding the date of its
transfer.
3. Which of the following transactions shall not be regarded as transfer as per the provisions
of section 47:
(A) Any distribution of capital assets on the total or partial partition of a Hindu Undivided
Family.
(B) Any transfer of a capital asset by a company to its subsidiary company, if the parent
company or its nominees hold the whole of the share capital of the subsidiary company,
and the subsidiary company is an Indian company.
(C) Any transfer, in a scheme of amalgamation, of a capital asset by the amalgamating
company to the amalgamated company if the amalgamated company is an Indian
company.
(D) All of the above.

D;

Solution: All of the following transactions shall not be regarded as transfer as per provisions
of Section 47:

1. Any distribution of capital assets on the total or partial partition of a Hindu Undivided
Family.

2. Any transfer of a capital asset by a company to its subsidiary company, If the parent
company or its nominees hold the whole of the share capital of the subsidiary company,
and the subsidiary company is an Indian company.

3. Any transfer, in a scheme of amalgamation, of a capital asset by the amalgamating


company to the amalgamated company if the amalgamated company is an Indian company.

4. Ms. Smita inherited a vacant site land consequent to the demise of her father on 10 th June,
2010. The land was acquired by her father on 10th April, 1990 for ₹ 40,000. The fair market
value of the land on 1st April, 2001 was ₹ 2,55,000 and on the date of inheritance, i.e., 10th
June, 2010 was ₹ 3,50,000. The cost of acquisition for Ms. Smita is —
(A) ₹ 40,000
(B) Nil
(C) ₹ 2,55,000
(D) ₹ 3,50,000

C;

Solution: Since the vacant site has been obtained under inheritance i.e. one of the modes
specified under section 49(1), therefore, its cost shall be the cost to the previous owner
(father) i.e. FMV as on 1-4-2001 i.e. ₹ 2,55,000.

5. Compute the capital gains for assessment year 2021-22 if Mr. Roy sells his personal
motorcar on 11-4-2020 for ₹ 2,55,000, which was acquired on 31-1-2017 for ₹ 6,50,000. The
expenses on transfer are 2% of selling price.
(A) Nil
(B) 3,95,000
(C) ₹ 3,82,000
(D) - ₹ 4,00,100

A;

Solution: Personal motorcar sold by Mr. Roy is a personal effect. Therefore, it was not a
capital asset and hence, no capital gains will arise.

6. Mr. Sunil sells the goodwill on 20-01-2021 for ₹ 38,00,000. It was self-generated by him and
he incurred cost of improvement thereof for ₹ 5,55,000 on 1-4-2016. Compute his taxable
gains. (Cost inflation index for F.Y. 2015-16 = 254 and 2020- 21 = 301)
(A) ₹ 38,00,000
(B) ₹ 32,45,000
(C) Nil
(D) ₹ 31,68,524

A;

Solution: In case of self-generated goodwill cost of acquisition and cost of improvement


shall be taken as nil. Computation of capital gains of Mr. Sunil (amounts in r)—

Particulars Self-Generated Goodwill

Full value of consideration 38,00,000

Less: Indexed cost of acquisition Nil

Less: Indexed cost of improvement Nil

Long-term capital gains 38,00,000

7. Under section 54EC, capital gains on transfer of land or building or both are exempted if
invested in the bonds issued by NHAI & RECL or Other notified bond—
(A) within a period of 6 months from the date of transfer of the asset.
(B) within a period of 6 months from the end of the relevant previous year.
(C) within a period of 6 months from the end of the previous year or the due date for filing
the return of income under section 139(1), whichever is earlier.
(D) At any time before the end of the relevant previous year.

A;

Solution: Under section 54EC, capital gains on transfer of land or building or both are
exempted if invested in the bonds issued by NHAI & RECL or other notified bond within a
period of 6 months from the date of transfer of the asset.
8. A owns a house property which was purchased by him on 1-5-1999 for ₹ 14,00,000. The said
property was destroyed by fire on 3-4-2020 and A received a sum of ₹ 52,00,000 from the
insurance company during the year. The market value of the above property as on 1-4-2001
was ₹ 17,00,000. Compute the capital gain for the assessment year 2021-22. (Cost inflation
index for F.Y. 2001-02= 100, and 2020-21 = 301)
(A) ₹ 9,54,000
(B) ₹ 83,000
(C) ₹ 35,00,000
(D) ₹ 38,00,000

B;

Solution: Computation of Capital gains—

Particulars ₹

Full value of consideration [Compensation so received] 52,00,000

Less: Indexed cost of acquisition (₹ 17,00,000 x 301/100) 51,17,000

Long term capital gains 83,000

9. Mr. X has sold his land for a consideration of ₹ 25,00,000 to Mr. Y. on 15-04-2020 . Mr. Y has
paid stamp duty of ₹ 3,00,000 @ 10% of stamp value. The said land was acquired by Mr. X
on 15-07-2018 for ₹ 12 lakhs. Mr. X was not satisfied with the stamp value determined by
stamp valuation authority and his case was referred to Valuation officer. The valuation
officer determined the value ₹ 35 lakhs. The taxable capital gains in hands of Mr. X will be –
(A) ₹ 23,00,000
(B) ₹ 14,00,000
(C) ₹ 10,00,000
(D) ₹ 18,00,000
D;

Solution: Computation of Capital gains—

Particulars ₹

Full value of consideration (Value determined by Valuation officer exceeds 30,00,000


SDV, hence SDV will be taken]

Less: Cost of acquisition 12,00,000

Short term capital gains 18,00,000

10. The exemption under section 54EC is withdrawn if the transfer of new asset, conversion
thereof in money or taking loan or advance on its security within years from the date of its
acquisition.
(A) 1
(B) 2
(C) 3
(D) 4

C;

Solution: The exemption under section 54EC is withdrawn if the transfer of new asset,
conversion thereof in money or taking loan or advance on its security within three years
from the date of its.
Clubbing of income set offs

1. X transfers his house property to a trust for benefit of Y till his death. In this case, till death
of Y, the income from house property shall be taxable in the hands of and afterwards in the
hands of
(A) X, Y
(B) X, legal heirs of Y
(C) Y, X
(D) Y, legal heirs of Y

D;

Solution: X transfers his house property to a trust for benefit of Y till his death. In this case,
till death of V, the income from house property shall be taxable in the hands of Y and
afterwards in the hands of X.

2. Rohit (a Chartered Accountant) is working as Accounts Officer in Raj (P) Ltd. on a salary of ?
20,000 p.m. He got married to Ms. Pooja who holds 25% shares of this company. What will
be the impact of salary paid to Rohit by the company in the hands of Ms. Pooja —
(A) 100% salary to be clubbed
(B) 50% salary to be clubbed
(C) No amount be clubbed
(D) 25% salary be clubbed.

C;

Solution: Clubbing provision shall not apply in relation to any income arising to the spouse
where the spouse possesses technical or professional qualifications and the income is solely
attributable to the application of his or her technical or professional knowledge and
experience. Salary paid to Rohit by the company shall not be clubbed.

3. Exemption of will be available u/s 10(32) to the parent in respect of minor's


income clubbed with the parent.
(A) ₹ 1,500
(B) ₹ 5,000
(C) ₹ 15,000
(D) ₹ 1,00,000 Your Answer :

A;

Solution: Exemption of ₹ 1,500 will be available u/s 10(32) to the parent in respect of
minor's income clubbed with the parent.

4. Mr. X transfers income of Z 51,000 from rent to his major son without transfer of house
property. Rent of ₹ 51,000 is —
(A) taxable in the hands of the transferor-father.
(B) taxable in the hands of the his son.
(C) taxable in the hands of the that parent whose total income is higher.
(D) exempt from tax.

A;

Solution: According to Section 60, If any person transfers the income from any asset
without transferring the asset itself, such income is to be included in the total income of the
transferor. Rent of ₹ 51,000 is taxable in the hands of the transferor-father.
5. During the P.Y. 2020-21, Mr. C has the following income and the brought forward losses:

Particulars ₹

Short term capital gains on sale of shares 1,50,000

Long term capital loss of A.Y. 2019-20 (96,000)

Short term capital loss of A.Y. 2020-21 (37,000)

Long term capital pin 75,000

What Is the capital gain taxable In the hands of Mr. C for the assessment year 2021-227

(A) ₹ 92,000
(B) ₹ 1,13,000
(C) ₹ 2,25,000
(D) ₹ 7 75,000

B;

Solution: Long Term Capital loss can be set off only against long-term capital gains and
cannot be set off against short term capital gains:

Particulars ₹ ₹

Short term capital gains on sale of shares 1,50,000

Less: Short term capital loss (37,000) 1,13,000

Long term capital gains 75,000

Less: Long term capital loss of A.Y. 2019-20 (Balance LTCL amounting (75,000) Nil
₹ 21,000 shall be carried forward)
Taxable Capital gains 1,13,000

6. Unexplained is to tax cash credits chargeable —


(A) @ 30%
(B) @ 15%
(C) @ 25%
(D) @ 78%

D;

Solution: As per Section 115BBE, the cash credits unexplained money, investment,
expenditure, etc. deemed as income under section 68 or section 69 or section 69A or
section 69B or section 69C or section 69D as reflected in the return or as determined by the
Assessing Officer not reflected in the return shall be taxable at rate of 60%. Tax is to be
increased by surcharge @ 25% of such tax and HEC @ 4% of such tax and surcharge. The
effective rate of tax under section 11513BE would, therefore, be 78% of such undisclosed
income etc., if declared in the return of income.

7. Loss from house property and losses in speculation business can be carried forward
respectively for —
(A) 8 Years and 4 Years
(B) 4 Years and 8 Years
(C) 8 Years and 8 Years
(D) 4 Years and 4 Years

A;
Solution: The loss from house property is allowed to be carried forward upto 8 assessment
years immediately succeeding the assessment year in which the loss was first computed.
The loss in speculation business can be carried forward only for a maximum period of 4
years from the end of the relevant assessment year in respect of which the loss was
computed.

8. Mr. Hussey for the previous year has (i) business loss of ₹ 1,30,000; (ii) income from salary ₹
2,40,000; and (iii) speculation gain of ₹ 1,10,000. His total income for income tax
assessment is :
(A) ₹ 3,50,000
(B) ₹ 2,20,000
(C) ₹ 2,40,000
(D) ₹ 1,10,000

C;

Solution: Computation of total income —

Particulars ₹ ₹

Salaries :

Income from salary (Computed) 2,40,000

Profit and gains of business or profession :

Speculation Gains 1,10,000

General business loss cannot be set-off from speculation gains and 1,10,000 -
balance loss of ₹ 20,000 shall be carried forward as it cannot be set-
off from salaries
Total Income 2,40,000

9. Mathur Storage (P) Ltd. engaged in specified business of cold storage chain has brought
forward business loss of ₹ 12 lakhs relating to assessment year 2020-21. During the previous
year 2020-21, its income from the said business is ₹ 9 lakhs. It also has profit from trade in
food grains of ₹ 6 lakhs. The total income of the company for the assessment year 2021-22
is :
(A) ₹ 15 lakhs
(B) ₹ 9 lakhs
(C) ₹ 6 lakhs
(D) ₹ 3 lakhs

C;

Solution: Computation of total income —

Particulars ₹ ₹

Profit and gains of business or profession :

General business profits 6,00,000

Specified business profits 9,00,000 -

Less: Brought forward specified business loss to be set off from 9,00,000 6,00,000
specified business gains and balance loss of ₹ 3,00,000 to be carried
forward to be set-off from specified business profits.

Total Income 6,00,000


10. Mr. RahuI have income from cloth business ₹ 1,00,000; Loss from agriculture ₹ 50,000;
Long-term capital gain ₹ 60,000 and short-term capital loss ₹ 80,000 find out his gross total
Income for assessment year 2021-22.
(A) ₹ 50,000
(B) ₹ 1,00,000
(C) ₹ 80,000
(D) ₹ 30,000

B;

Solution: Loss from agriculture cannot be setoff, since agricultural income is exempt from
tax. Computation of Gross total income—

Particulars ₹ ₹

Capital Gains

Long Term capital gains 60,000

Less: Short term capital loss [to be set off from LTCG and balance of ₹ 60,000 -
20,000 shall be carried forward]

Profit and gains of business or profession:

General business profits from cloth business 1,00,000

Gross Total Income 1,00,000


Deductions from gross total income

1. Which of the following deductions are not allowed from the gross total income of the
assessee if the assessee does not furnish the return of income as per the due dates
specified under Section 139(1) of the Act.
(A) Deduction in respect of payments - 80C to 80GGC
(B) Deduction in respect of certain Incomes - 80IA to 8ORRB
(C) Other deductions - Section 80U
(D) All of the above

B;

Solution: Section 80AC stipulates compulsory filing of return of income on or before the due
date specified under section 139(1), as a pre-condition for availing benefit of deductions
under any provision of Chapter VI-A under the heading "C.

— Deductions in respect of certain incomes" i.e. Section 80IA to Section 8ORRB.

2. The maximum amount of deduction under section 80C is –


(A) ₹ 70,000
(B) ₹ 1,50,000
(C) ₹ 50,000
(D) ₹ 2,00,000

B;

Solution: The maximum permissible deduction under section 80C is ₹ 1,50,000.


3. An Individual assessee, resident in India, has made the following deposit/payment during
the previous year 2020-21:

Particulars ₹

Contribution to the public provident fund 1,15,000

Insurance premium paid on the life of the spouse (policy taken on 1- 4-2015) 25,000
(Assured value ₹ 2,00,000)

What is the deduction allowable under section 80C for A.Y. 2021-22?

(A) ₹ 1,15,000
(B) ₹ 1,40,000
(C) ₹ 1,35,000
(D) ₹ 2,00,000

C;

Solution: The amount of deduction admissible u/s 80C will be —

Particulars ₹

Contribution to the public provident fund 1,15,000

Insurance premium paid on the life of the spouse (policy taken on 1- 4-2014) 20,000
(Assured value ₹ 2,00,000, it will be restricted to 10% of capital sum assured)

Total admissible deduction 1,35,000

4. In case of non-salaried employee, the amount of deduction under Section 80CCD shall not
exceed
(A) Sum paid/deposited by assessee to the credit of his account in the notified pension
scheme, or 20% of his gross total income in the previous year, whichever is less.
(B) Sum paid/deposited by assessee to the credit of his account in the notified pension
scheme, or 15% of his gross total income in the previous year, whichever is less.
(C) Sum paid/deposited by assessee to the credit of his account in the notified pension
scheme, or 10% of his business income in the previous year, whichever is less.
(D) Sum paid/deposited by assessee to the credit of his account in the notified pension
scheme, or 15% of his business income in the previous year, whichever is less.

A;

Solution: In case of non-salaried employee, the amount of deduction under Section 80CCD
shall not exceed sum paid/deposited by assessee to the credit of his account in the notified
pension scheme, or 20% of his gross total income in the previous year, whichever is less.

5. The deduction in respect of health insurance premia under section 80D is allowed to –
(A) Individual
(B) Both Individual and HUF
(C) HUF
(D) None of these

B;

Solution: The deduction in respect of health premia under section 80D is allowed to Both
Individual and HUF

6. Rajan paid ₹ 25,000 to LIC of India for the maintenance of his disabled son and incurred ₹
15,000 for the treatment of his handicapped wife who is working in State Bank of India. The
deduction allowable to him u/s 80DD is —
(A) ₹ 15,000
(B) ₹ 25,000
(C) ₹ 50,000
(D) ₹ 75,000

D;

Solution: The quantum of deduction under Section 80DD is ₹ 75,000 and in case of severe
disability (i.e. person with 80% or more disability) the deduction shall be ₹ 1,25,000
irrespective of the amount of expenditure so incurred.

7. Sahil works in a technology company. On 1st January, 2018, he took a loan of ₹ 2,40,000
from his company for the education of his daughter. During the year 2020-21, he paid an
interest of ₹ 26,000 towards the said loan and repaid principal component of ₹ 10,000. The
deduction that he can claim under section 80E would be —
(A) Nil
(B) ₹ 24,000
(C) ₹ 46,000
(D) ₹ 10,000

A;

Solution: Under Section 80E, Assessee shall be eligible for deduction in respect of any
amount paid by him in the previous year, out of his income chargeable to tax, by way of
interest on loan taken by him from any financial institution or any approved charitable
institution for the purpose of pursuing

(i) his higher education, or

(ii) for the higher education of his relative. Thus, deduction admissible will be nil, since loan
is taken from employer.
8. Mr. Rajeev whose adjusted gross total income is ₹ 1,90,000 pays by account payee cheque
₹21,000 as donation to Prime Ministers Drought Relief Fund. The amount of deduction
available to him is —
(A) ₹ 21,000
(B) ₹ 19,000
(C) ₹ 10,500
(D) ₹ 9,500

C;

Solution: Computation of amount of deduction u/s 80G :

% of donation
Amount of Amount of
Donation to eligible for
donation (₹) deduction (₹)
deduction

Prime Ministers Drought Relief


21,000 50% 10,500
Fund — by cheque

9. Deduction under section 80GG in respect of rent paid is not available to —


(A) Individual
(B) Salaried employee who is in receipt of any HRA from his employer.
(C) Salaried employee, who is in receipt of house facility from his employer.
(D) Both (B) and (C)

B;

Solution: Deduction u/s 80GG in respect of rent paid is not available to salaried employee
who is in receipt of any HRA from his employer.
10. Mr. Shiva has gross total income of ₹ 3,75,000. He has given the following donations :
 National Children's Fund ₹ 25,000 — by cheque
 Prime Minister's Drought Relief Fund ₹ 30,000 — by cheque
 National Blood Transfusion Council ₹ 40,000 — by cash
 National Illness Assistance Found ₹ 20,000 — (₹ 2,000 by cash and balance by cheque)

Compute the amount deductible under section 80G.

(A) ₹ 60,000
(B) ₹ 1,00,000
(C) ₹ 40,000
(D) ₹ 37,500

A;

Solution: Computation of amount of deduction u/s 80G:

Donation to Amount of % of donation Amount of deduction


donation (₹) eligible for (₹)
deduction

National Children's 25,000 100% 25,000


Fund — by cheque

Prime Minister's 30,000 50% 15,000


Drought Relief Fund —
by cheque

National Blood 40,000 100% Nil (Cash donation in


Transfusion Council — excess of ₹ 2,000 would
by cash not qualify for
deduction)
National Illness 20,000 100% ₹ 20,000 (The whole
Assistance Fund — ₹ amount qualifies for
2,000 by cash and deduction, since cash
balance by cheque donation in this case
does not exceed t 2,000)

Total admissible deductions 60,000


Income from house property

1. Vidya received ₹ 90,000 in May, 2020 towards recovery of unrealised rent, which was
deducted from actual rent during the P.Y. 2018-19 for determining annual value. Legal
expense incurred in relation to unrealized rent is ₹ 20,000. The amount taxable under
section 25A for A.Y. 2021-22 would be -
(A) ₹ 70,000
(B) ₹ 63,000
(C) ₹ 43,000
(D) ₹ 49,000

B;

Solution: Computation of amount taxable u/s 25A for A.Y. 2021-22 :

Unrealised rent received 90,000

Less: Deduction @ 30% 27,000

Amount taxable under Section 25A 63,000

2. In case any property is owned by an assessee and the same is given by him to the
partnership firm, in which he is a partner, for carrying on the business of such firm, then the
income from such property will :
(A) Not be taxable
(B) be taxable as Income from Other Sources
(C) be taxable as Income from house property
(D) be taxable as Profits and gains of business
A;

Solution: In case any property is owned by an assessee and the same is given by him to the
partnership firm, in which he is a partner, for carrying on the business of such firm, then it
will be treated as if the property is used by the assessee for his own business and thus, the
income from such property will not be taxable under Income from house property.

3. In which of the following cases the annual value of the house is taken to be NIL.
(i) One or two self occupied house.
(ii) House remaining vacant for the part of the period and let out for the remaining period.
(iii) House remaining vacant throughout the previous year owing to business and profession
carried outside that place.
(A) (i) and (ii)
(B) (i) and (iii)
(C) (ii) and (iii)
(D) (ii)

B;

Solution: Section 23 provides relief to the taxpayer by allowing him an option to claim nil
annual value in respect of any two houses, declared as self-occupied, instead of one such
house as provided earlier. Where the property is self-occupied for own residence or
unoccupied throughout the previous year, its Annual Value will be Nil, provided no other
benefit is derived by the owner from such property. The expression "Unoccupied property"
refers to a property which cannot be occupied by the owner by reason of his employment,
business or profession at a different place and he resides at such other place in a building
not belonging to him.
4. Sejal is the owner of a house property covered under the Rent Control Act. Municipal value
₹ 3,00,000, Actual rent ₹ 2,80,000, Fair rent ₹ 3,60,000 and Standard rent is ₹ 2,50,000. The
gross annual value of the house property will be
(A) ₹ 3,50,000
(B) ₹ 2,50,000
(C) ₹ 3,60,000
(D) ₹ 2,80,000

D;

Solution: Computation of Gross annual value —

Particulars House

Expected Rent [Municipal value ₹ 3,00,000 or Fair rent ₹ 3,60,000 whichever 2,50,000
is higher subject to maximum of Standard rent ₹ 2,50,000]

Actual rent receivable 2,80,000

Gross Annual value (Higher of the above two) 2,80,000

5. Prem owns a house in Madras. During the previous year 2020-21, 2/3rd portion of the house
was self-occupied and 1/3rd portion was let out for residential purposes at a rent of ₹ 8,000
p.m. Municipal value of the property is ₹ 3,00,000 p.a., fair rent is ₹ 2,70,000 p.a. and
standard rent is ₹ 3,30,000 p.a. He paid municipal taxes @10% of municipal value during the
year. A loan of ₹ 25,00,000 was taken by him during the year 2015 for acquiring the
property. Interest on loan paid during the previous year 2020-21 was ₹ 1,20,000. Compute
Prem's income from house property for the A.Y. 2021-22.
(A) (₹ 57,000)
(B) ₹ 23,000
(C) ₹ 80,000
(D) (₹ 59,800)

A;

Solution:

Self-occupied (2/3rd portion):

Net Annual Value Nil

Less: Interest on the borrowed capital u/s 24(b) [₹ 1,20,000 × 2/3] 20

Net Income from Self-occupied portion -80,000

Let-out (1/3rd portion):

Expected Rent [Municipal value ₹ 1,00,000 or Fair rent ₹ 90,000 whichever is 1,00,000
higher subject to maximum of Standard rent ₹ 1,10,000]

Actual rent received (₹ 8,000 × 12) 96,000

Gross Annual Value 1,00,000

Less: Municipal taxes paid (10% of ₹ 1,00,000) 10,000

Net Annual Value 90,000

Less: Deductions u/s 24 –

(1) Statutory deduction @ 30% of NAV 27,000

(2) Interest on the borrowed capital [₹ 1,20,000/3] 40,000

Net Income from let-out portion 23,000


Thus, total Income from House Property = (₹) - 80,000 + ₹ 23,000 = ₹ (57,000).

6. S owns a house, which is self-occupied upto 31-5-2020. W.e.f. 1-6-2020, the property is let
to R at ₹ 42,000 p.m. Determine the Gross Annual Value of the house if the municipal value
is ₹ 4,15,000; Fair Rent ₹ 4,30,000 and standard rent is ₹ 4,10,000.
(A) ₹ 4,20,000
(B) ₹ 4,30,000
(C) ₹ 4,10,000
(D) ₹ 4,15,000

A;

Solution: Computation of Gross annual value —

Particulars ₹

Expected Rent [Municipal value ₹ 4,15,000 or Fair rent ₹ 4,20,000 whichever is 4,10,000
higher subject to maximum of Standard rent ₹ 4,10,000]

Actual rent received (₹ 42,000 x 10) 4,20,000

Gross Annual Value 4,20,000

7. Which of the following deductions are to be made from annual value to compute Income
from House Property?
(A) Statutory deduction
(B) Interest on borrowed capital
(C) Both (A) and (B)
(D) None of the above
C;

Solution: Standard deduction and interest on borrowed capital is allowed as deduction


while computing income from house property.

8. The maximum limit of deduction under section 24(b) for interest on borrowed capital
before 1-4-1999 for construction of house property used for self-occupation is:
(A) ₹ 30,000
(B) ₹ 2,00,000
(C) ₹ 50,000
(D) ₹ 60,000

A;

Solution: The maximum limit of deduction under section 24(b) for interest on borrowed
capital before 1-4-1999 for construction of house property used for self-occupation is ₹
30,000.

9. R took a loan of ₹ 8,00,000 on 1-4-2019 from a bank for construction of a house. The loan
carries an interest @ 12% p.a. The construction is completed on 31-03-2020. The entire loan
along with interest is still outstanding on 31-03-2021. The pre-construction period interest
will be
(A) ₹ 96,000
(B) Nil
(C) ₹ 1,92,000
(D) ₹ 1,50,000

B;
Solution: In this there is no pre-construction period since the house is completed in the year
of borrowing, hence, pre-construction period interest shall be NIL.

10. M took a loan of ₹ 6,00,000 on 1-4-2018 from his relative for construction of a house. The
loan carries an interest @ 10% p.a. The construction is completed on 15-6-2020. The entire
loan and interest is still outstanding. Compute the interest allowable for the assessment
year 2021-22.
(A) ₹ 60,000
(B) ₹ 1,80,000
(C) ₹ 84,000
(D) ₹ 24,000

C;

Solution: The pre-construction period will commence from 01-04-2018 and end on 31-03-
2020 since the property is completed on 15-06-2020. [₹ 6,00,000 x 10% x 2] = ₹ 1,20,000. It
will be allowed in 5 equal installment. ₹ 1,20,000 ÷ 5 = ₹ 24,000. Total allowable interest is
as under –

Particulars ₹

Pre-construction period interest 24,000

Interest for F.Y. 2020-21 (₹ 6,00,000 x 12% x 1) 60,000

Total allowable interest 84,000

11. Nisha has two houses, both of which are self occupied. The particulars of these are given
below :
Particulars Value in ₹

House — I House — II

Municipal Valuation per annum 1,20,000 1,15,000

Fair Rent per annum 1,50,000 1,75,000

Standard rent per annum 1,00,000 1,65,000

Date of completion 31-03-2018 31-03-2019

Municipal taxes payable during the year (paid for 12% 8%


House II only)

Interest on money borrowed for acquisition of house 1,80,000 80,000


properties

Compute Nisha's income from the House Property for the Assessment Year 2021-22.

(A) ₹ -2,60,000
(B) ₹ -2,00,000
(C) ₹ -1,80,000
(D) ₹ -30,000

B;

Solution: Section 23 of the Income-tax Act has been amended so as to provide relief to the
taxpayer by allowing him an option to claim nil annual value in respect of any two houses,
declared as self-occupied, instead of one such house as provided earlier. Besides this
section 24 of the Income-tax Act has been amended to provide that the monetary limit of
deduction on account of interest payable on borrowed capital i.e. ₹ 30,000 / ₹ 2,00,000
shall continue to apply to the aggregate of the amounts of deduction in case of more than
one self-occupied houses. Hence, the Income from house property shall be computed as
under:

Computation of income from houses when they are treated as self-occupied (amount in ₹)

Particulars House I and II

Annual Value [Since both the houses are self-occupied] Nil

Less: Interest on loans [ House — I : ₹ 1,80,000 and House — II : ₹ 2,00,000


80,000 (Restricted to 2,00,000)]

Income from house property -2,00,000

12. Where property consisting of any buildings or lands appurtenant thereto is held as stock-in-
trade and the whole or any part of the property is not let out during the whole or any part
of the previous year, the annual value of such property or part of the property for the
period from the end of the financial year in which certificate of completion of
construction of the property is obtained from the competent authority shall be taken as
"Nil".
(A) One year
(B) Two years
(C) Three years
(D) Five years

B;

Solution: Where property consisting of any buildings or lands appurtenant thereto is held as
stock-in-trade and the whole or any part of the property is not let out during the whole or
any part of the previous year, the annual value of such property or part of the property for
the period of two years from the end of the financial year in which certificate of completion
of construction of the property is obtained from the competent authority shall be taken as
"Nil".
Income from Other Sources

1. Under the Income-tax Act, 1961, dividend derived from the shares held as stock-in-trade are
taxable under the head —
(A) Capital gains
(B) Income from other sources
(C) Profits and gains of business or profession
(D) Either capital gains or Profits and gains of business or profession.

A;

Solution: Dividend derived from the shares held as stock-in-trade is taxable under the head
Income from other sources.

2. A private limited company engaged in manufacturing activity had general reserve of ₹ 20


lakh. It granted a loan of ₹ 5 lakh to a director who held 13% shareholding cum voting rights
in the company. The said loan was re-paid by him before the end of the year. The amount of
deemed dividend arising out of the above transaction is—
(A) ₹ 2,60,000
(B) ₹ 2,40,000
(C) ₹ 5,00,000
(D) Nil

C;

Solution: Dividend under Section 2(22)(e) includes any payment made by a company, not
being a company in which the public are substantially interested, of any sum by way of
advance or loan, to the extent of accumulated profits (capitalised accumulated profits not
included here),—

(i) to a shareholder who is the beneficial owner of equity shares holding not less than 10%
of the voting power.
(ii) to any concern (HUF/ Firm/ AOP/ 801/ Company) in which the shareholder referred to in
(i) above is a member or a partner and in which he has a substantial interest.

Thus, ₹ 5,00,000 shall be regarded as deemed dividend and taxable in hands of shareholder.

3. Dividend received from a foreign company (net) ₹ 8,000. Nothing has been paid to the
Government of India out of tax deducted at source. Calculate the amount to be included
while computing income under the head 'Income from Other Sources'.
(A) Nil
(B) ₹ 8,000
(C) ₹ 11,428
(D) ₹ 2,400

B;

Solution: The amount of dividend to be included in computing the total income in this case
will be ₹ 8,000.

4. Seema received rent of ₹ 7,00,000 from letting of plant and machinery (letting of plant and
machinery is not taxable under Profit and gain from Business and profession). She expended
₹ 40,000 for repairs of machinery but failed to deduct TDS on such expense u/s 194C.
Calculate the taxable income in her hand.
(A) ₹ 7,00,000
(B) ₹ 6,60,000
(C) ₹ 6,72,000
(D) ₹ 6,50,000

C;

Solution: Computation of Income from other Sources —

Particulars ₹ ₹

Income from other sources

Rent of machinery 7,00,000

Less: Repairs [70% of ₹ 40,000] (30% of the expenditure shall be 28,000 6,72,000
disallowed as tax has not been deducted at source]

5. Compute income taxable under head income from other sources:

Particulars ₹

(i) Interest on bank deposits 3,000

(ii) Winnings from lotteries (net) 33,936

(iii) Interest on Post office savings bank account 500

(A) ₹ 51,480
(B) ₹ 51,980
(C) ₹ 36,936
(D) ₹ 37,436

A;
Solution: Computation of income from other sources —

Particulars ₹

(i) Interest on bank deposits 3,000

(ii) Winnings from lotteries [Gross] [₹ 33,936 x 100 + 70] 48,480

(iii) Interest on Post office savings bank account [Exempt u/s 10(35) Exempt

Income from other sources 51,480

6. Mrs. X received the following gifts during the year. Which gifts shall be included in
computing the income from other sources?
(A) Gift of ₹ 2,000 from her employer.
(B) Gift of ₹ 51,000 on 15th December from her mother's friend.
(C) Gift of ₹ 1,21,000 from her husband's brother.
(D) Gift of ₹ 60,000 on 25th November from her father's brother.

B;

Solution: Mother's friend is not included in the ambit of relative, hence gift received from
her will be taxable under income from other sources.

7. On 30th December, 2020, Raju gets by gift a commercial flat from the elder brother of his
father (stamp duty value is ₹ 25,00,000). The amount chargeable to tax in the hands of Raju
is —
(A) ₹ 25,00,000
(B) ₹ 24,50,000
(C) ₹ 20,00,000
(D) Nil

D;

Solution: Brother or sister of either of the parents of the individual fall under the ambit of
relative. Hence, gift received from them shall not be taxable.

8. Ashok took possession of property on 31st August 2020 booked by him three years back at ₹
25 lakhs, The Stamp Duty Value (S0V) of the property as on 31st August 2020 was ₹ 31 lakh
and on date of booking it was ₹ 29 lakh. He had paid ₹ 2 lakh by A/c payee cheque as down
payment on date of booking. Which of the following will be considered as income, if any,
and in which previous year—
(A) ₹ 4 lakhs in P.Y. 2020-21
(B) ₹ 4 lakhs in P.Y. 2016-17
(C) ₹ 6 lakhs in P.Y. 2020-21
(D) No Income shall be taxable, since down payment was made by A/c cheque while
booking the property.

A;

Solution: The difference between the stamp duty value at the time of agreement and the
consideration paid shall be taxable as Income from other sources when the possession of
the property Is given. Hence, the amount of [₹ 29,00,000—₹ 25,00,000] = ₹ 4,00,000 shall
be taxable In FY 2020-21.

9. Sameer received the following income during financial year 2020-21 : Director's fees ₹
5,000, income from agricultural land in Pakistan ₹ 15,000, rent from let-out of land in Jaipur
₹ 20,000, interest on deposit with HDFC Bank ₹ 1,000 and dividend from Indian company ₹
5,000. His income from other sources is —
(A) ₹ 41,000
(B) ₹ 46,000
(C) ₹ 31,000
(D) ₹ 26,000

B;

Solution: Computation of the Income under the head "Income from other Sources" —

Particulars ₹

1. Directors Fee from a Company 5,000

2. Income from Agricultural land in Pakistan 15,000

3. Rent from let out land 20,000

4. Interest on deposit of banks 1,000

5. Dividend from Indian company 5,000

Income from other sources 46,000

10. "Specified foreign company" means :


(A) Foreign company in which Indian company holds 26% or more in nominal value of the
equity share capital of the company.
(B) Foreign company in which Indian company holds 51% or more in nominal value of the
equity share capital of the company.
(C) Company which is registered in India.
(D) None of these.
A;

Solution: "Specified foreign company" means Foreign company in which Indian company
holds 26% or more in nominal value of the equity share capital of the company.
IT Authorities Procedure

1. In issuing the directions or orders of Jurisdiction of Income Tax authorities, the CBDT may
have regard to which of the under-mentioned criteria:
(A) territorial area
(B) incomes or classes of income;
(C) persons or classes of persons
(D) All of the above

D;

Solution: As per Section 120(3), in issuing the directions or orders of Jurisdiction of Income
Tax authorities, the CBDT or other authorised income-tax authority may have regard to any
one or more of the following criteria, namely —

(a) territorial area;

(b) persons or classes of persons;

(c) incomes or classes of income; and

(d) cases or classes of cases.

2. In case of survey the books of accounts cannot be impounded for a period exceeding days
(exclusive of holidays), without obtaining prior approval of Chief Commissioner of Income
Tax.
(A) 10
(B) 15
(C) 30
(D) 60

B;

Solution: In case of survey the books of accounts cannot be impounded for a period
exceeding 15 days (exclusive of holidays), without obtaining prior approval of Chief
Commissioner of Income Tax.

3. Where a person required to furnish a return of income under section 139, fails to do so
within the time prescribed in Section 139(1), he shall pay, by way of fee, a sum of, if the
return is furnished after 31st day of December of the assessment year and his total income
does not exceed ₹ 5,00,000.
(A) ₹ 100 for every day during which such failure continues.
(B) ₹ 5,000
(C) ₹ 1,000
(D) ₹ 10,000

C;

Solution: Where a person required to furnish a return of income under section 139, fails to
do so within the time prescribed in Section 139(1), he shall pay, by way of fee, a sum of, ₹
1,000 if the return is furnished after 31 st day of December of the assessment year and his
total income does not exceed ₹ 5,00,000.

4. In the case of an individual assessee, the return of income must be verified by following,
except —
(A) Individual himself.
(B) Where he is absent from India, by some person duly authorised by him in this behalf.
(C) Where he is mentally incapacitated from attending to his affairs, by his guardian or any
other person competent to act on his behalf.
(D) Spouse.

D;

Solution: In the case of an individual assessee, the return of income must be verified by
following, —

• Individual himself.
• Where he is absent from India, by some person duly authorised by him in this behalf.
• Where he is mentally incapacitated from attending to his affairs, by his guardian or any
other person competent to act on his behalf.

Thus, spouse cannot verify the return of the individual.

5. Quoting of PAN is mandatory when a person is entering into following transactions :


(1) Sale of immovable property of ₹ 10 lakh or more
(2) Deposit of cash exceeding ₹ 50,000 in Post Office Savings Bank
(3) Deposit of cash aggregating ₹ 40,000 in one day in a bank
(4) Contract of sale and purchase of securities exceeding ₹ 1 Iakh

Select the correct answer from the options given below —

(A) (1), (2) and (3)


(B) (1), (2) and (4)
(C) (1), (3) and (4)
(D) (1), (2), (3) and (4)

B;
Solution: Quoting of PAN is mandatory when a person is entering into following
transactions:

(1) Sale of immovable property of ₹ 10 Iakh or more

(2) Deposit of cash exceeding ₹ 50,000 in Post Office Savings Bank

(3) Contract of sale and purchase of securities exceeding ₹ 1 Iakh

6. Under Section 142A of the Income tax Act, the Valuation Officer shall send a copy of his
estimate to the Assessing Officer and the assessee within a period of
(A) 6 months from the end of the month in which the reference is made to him.
(B) 6 months from the date in which the reference is made to him.
(C) One year from the end of the month in which the reference is made to him.
(D) One Year from the date in which the reference is made to him.

A;

Solution: The Valuation Officer shall send a copy of the report of the estimate made as
above to the Assessing Officer and the assessee, within a period of 6 months from the end
of the month in which a reference is made to him.

7. The notice under section 143(2) must be served within —


(A) 12 months from the date of filing of return
(B) 12 months from the due date of filing the return under section 139(1) or from the date
of filing of return of income
(C) 6 month from the end of the financial year in which the return was furnished
(D) 6 months from the end of month in which the return was furnished

C;
Solution: Scrutiny notice under Section 143(2) is required to be served (not merely issued)
before the expiry of 6 months from the end of the financial year in which the return is
furnished.

8. An apparent error in the assessment order passed u/s 143(3) dated 15-11-2020 was noticed
by the assessee in February, 2021. The time limit for seeking rectification of mistake is
available up to :
(A) 31-03-2025
(B) 31-03-2024
(C) 31-03-2022
(D) 31-03-2023

A;

Solution: No amendment under section 154 shall be made after the expiry of four years
from the end of the financial year in which the order sought to be amended was passed.
Thus, in this case rectification can be made upto 31-03-2025.

9. Mr. X assessment under section 143(1) completed for the assessment year 2015-16. Notice
under section 148 issued on 30-03-2020 and served on 04-04-2020. Mr. X income which has
escaped assessment is ₹ 80,000 and he has not made full or true disclosure. Discuss the
validity of notices issued u/s 148 for making assessment u/s 147?
(A) Invalid, since the notice is not served within 4 years from the end of the relevant
assessment year i.e. upto 31-03-2020
(B) Valid, since the notice is issued within 4 years from the end of the relevant assessment
year i.e. upto 31-03-2020
(C) Invalid, since the income escaping assessment is less than Z 1,00,000.
(D) Valid, since the notice is served within 6 years from the end of the relevant assessment
year.

B;

Solution:

Assessee A.Y. Notice u/s Full and Income Validity or otherwise


148 issued true escaping of the notice (with
on disclosure assessment reasons)
made (₹)

Mr. X 2015-16 30-03-2020 No 80,000 Valid : Since the


notice is issued within
4 years from the end
of the relevant A.Y. i.e.
upto 31-03-2020

10. Section 153D provides that assessment or reassessment of search cases in respect of each
assessment year referred to in section 153A(1)(b) or the assessment year referred to in
153B(1)(b) shall not be made by an Assessing Officer below the rank of Joint Commissioner
without the previous approval of the
(A) Joint Commissioner
(B) Additional Commissioner
(C) Chief Commissioner
(D) Commissioner

A;

Solution: Section 153D provides that assessment or reassessment of search cases in respect
of each assessment year referred to in section 153A(1)(b) or the assessment year referred
to in 153B(1)(b) shall not be made by an Assessing Officer below the rank of Joint
Commissioner without the previous approval of the Joint Commissioner.

11. If any person fails to give notice to the Assessing Officer of discontinuance of business or
profession, then he shall be liable to pay penalty equal to for every day during
which failure continues.
(A) ₹ 100
(B) ₹ 200
(C) ₹ 500
(D) ₹ 1,000

A;

Solution: If any person fails to give notice to the Assessing Officer of discontinuance of
business or profession, then he shall be liable to pay penalty equal to ₹ 100 for every day
during which failure continues.

12. Which of the following powers cannot be exercise by Income tax authority while carrying
out survey under Section 133A(2A), for the purpose of verifying that tax has been deducted
or collected at source in accordance with the provisions of Chapter XVII-B or Chapter XVII-
BB?
(A) It may require the deductor or the collector or any other person who may at the time
and place of survey be attending to such work to afford him the necessary facility to
inspect such books of account or other documents as he may require and which may be
available at such place
(B) It may place marks of identification on the books of account or other documents
inspected by him and take extracts and copies thereof;
(C) It may record the statement of any person which may be useful for, or relevant to, any
proceeding under the Act.
(D) It may impound and retain in his custody, any books of account or documents inspected
by him.

D;

Solution: Under Section 133A(2A), the income tax authority shall not impound and retain in
his custody any books of account or documents inspected by him.
Non Resident Tax, DTR

1. Sea Port Shipping Line, a non-resident foreign company operating its ships on the Indian
Ports during the previous year ended on 31-3-2021, had collected in India freight of ₹ 100
lakh, demurrages of ₹ 20 lakhs and handling charges of ₹ 10 Iakh. The expenses of operating
its fleet during the year for the Indian Ports were ₹ 110 Iakh. The deemed profits and gains
of business and profession as per provisions of Section 44B shall be —
(A) ₹ 9.75 Iakh
(B) ₹ 10 Iakh
(C) ₹ 6.5 Iakh
(D) Nil

A;

Solution: The provisions of Section 44B would apply in this case. Thus, business income shall
be computed as follows –

Such freight shall form the part of gross receipts. Accordingly, gross receipts = ₹ 100 Iakh + ₹
20 Iakh + ₹ 10 lakh = ₹ 130 Iakh. Therefore, profits and gains of business or profession =
7.5% of gross receipts i.e. 7.5% of ₹ 130 Iakh = ₹ 975,000.

2. Smith, a foreign national and a cricketer came to India as a member of Australian cricket
team in the year ended 31" March, 2021. He received t 5 Iakhs for participation in matches
in India. He also received T 1 lakh for an advertisement of a product on TV. He contributed
articles in a newspaper for which he received T 10,000. He has Incurred T 50,000 as
expenditure on earning such income in India. He has no other Income in India during the
year. Compute tax liability of Smith for assessment Year 2021-22.
(A) ₹ 1,16,480
(B) ₹ 25,480
(C) ₹ 1,26,880
(D) ₹ 1,90,320

C;

Solution: Income shall be computed as per provision of section 115BBA and no deduction in
respect of any expenditure shall be allowed from such Income.

Computation of tax liability —

Particulars ₹

Fees for participation in matches In India 5,00,000

Income from contributing articles In newspaper 10,000

Income from advertisement of a product on TV 1,00,000

Total Income 6,10,000

Tax liability:

U/s 115BBA @ 20% on ₹ 6,10,000 1,22,000

Add: HEC @ 4% 4880

Tax liability 1,26,880

3. Where the total income of an assessee, being a non-resident Indian includes income by way
of long-term capital gains arising from transfer of unlisted securities, applicable income-tax
rate on such income is —
(A) 10% plus surcharge (as applicable) plus HEC
(B) 20% plus surcharge (as applicable) plus HEC
(C) 30% plus surcharge (as applicable) plus HEC
(D) 40% plus surcharge (as applicable) plus HEC

A;

Solution: Where the total income of an assessee, being a non-resident Indian includes
income by way of long-term capital gains arising from transfer of unlisted securities,
applicable income-tax rate on such income is 10% plus surcharge (as applicable) plus HEC.

4. Where a non-resident/foreign company is liable to tax at special rate of 10% on account of


royalty or fee for technical services, such assessee shall —
(A) Be eligible for deduction of any expenses from such income u/s 28 to 44C or section 57.
(B) Not be eligible for any deduction under Chapter VI-A on account of such income.
(C) Neither be eligible for deduction of any expenses u/s 28 to 44C or section 57 nor be
eligible for any deduction under Chapter VI-A.
(D) Not be eligible for deduction of any expense under section 28 to 44C or section 57 but
shall be eligible for deduction under Chapter VI-A.

D;

Solution: Where a non-resident/foreign company is liable to tax at special rate of 10%


under section 115A on account of royalty or fee for technical services, such assessee shall
not be eligible for deduction of any expense under section 28 to 44C or section 57 but shall
be eligible for deduction under Chapter VI-A.

5. The tax on the income of non-resident can be/may be recovered : (1) By deduction of tax at
source (2) From his associates (3) From his agents Select the correct answer from the
options given below —
(A) (1) only
(B) (1) and (2) only
(C) (1), (2) and (3)
(D) None of the above

C;

Solution: The tax on the income of non-resident can be/may be recovered: (1) By deduction
of tax at source (2) From his associates (3) From his agents

6. Mr. Sam (aged 40 years), a US football match referee, has earned income from football
tournaments in India for assessment year 2021-22. What are the TDS provisions applicable
while making payment to him?
(A) TDS @ 20.8% as per section 194E
(B) TDS @ 5.2% as per section 194E
(C) TDS under section 195
(D) No tax is deductible at source

C;

Solution: "Match referee" would not fall within the meaning of "sportsmen" to attract the
provisions of Section 115BBA. Therefore, although the payments made to non-resident
"match referee" are "income" which has accrued and arisen in India, the same are not
taxable under the provisions of Section 115BBA. They are subject to the normal rates of tax.
Hence, tax will be deducted at source under Section 195 of the Act.

7. Interest income earned by a non-resident during the P.Y. 2020.21 on bonds, issued by ABC
Ltd., an Indian company, under a scheme notified by the Central Government, which were
purchased by him in convertible foreign currency, is —
(A) Taxable @ 10% plus surcharge (as applicable) plus HEC
(B) Taxable @ 15% plus surcharge (as applicable) plus HEC
(C) Taxable @ 20% plus surcharge (as applicable) plus HEC
(D) Not taxable

A;

Solution: Interest income earned by a non-resident on bonds, issued by an Indian company,


under a scheme notified by the Central Government, which were purchased by him in
convertible foreign currency, is taxable @ 10% plus surcharge (as applicable) plus HEC under
Section 115AC of the Income tax Act, 1961.

8. The aggregate participation or investment in the eligible investment fund, directly or


indirectly, by persons resident in India does not exceed of the corpus of the
fund.
(A) 2%
(B) 4%
(C) 5%
(D) 10%

C;

Solution: The aggregate participation or investment in the eligible investment fund, directly
or indirectly, by persons resident in India does not exceed 5% of the corpus of the fund.

9. XYZ Ltd., a foreign company, furnishes you with the following information—
(1) Interest received from Government of India from investment in government bonds in
foreign currency ₹ 5,00,000; expenditure incurred to earn such income ₹ 10,000.
(2) Interest received from infrastructure debt fund ₹ 6,25,000; expenditure incurred on
collection ₹ 25,000.
(3) Interest received from Indian concern from investment made in debentures in Indian
currency ₹ 4,85,000; expenditure incurred on account of collection ₹ 25,000.

You are required to determine the Total Income of the company.

(A) ₹ 15,50,000
(B) ₹ 15,85,000
(C) ₹ 15,95,000
(D) ₹ 16,10,000

B;

Solution: Computation of total income of the company

Particulars ₹ ₹

(1) Interest received from Indian Government from 5,00,000


investment in government bonds u/s 115A(1)(a)

Less: Expenditure incurred Nil 5,00,000

(2) Interest received from infrastructure debt fund 6,25,000


referred u/s 10(47)

Less: Expenditure incurred Nil 6,25,000

(3) Interest received from Indian concern from investment 4,85,000


in debentures in Indian currency

Less: Expenditure incurred 25,000 4,60,000

Total Income 15,85,000


10. Shipcargo Inc., a company based in Netherlands operating its ships to and fro Cochin port,
collected freight of ₹ 85 lakhs, demurrage of ₹ 5 lakhs and handling charges of ₹ 2 lakhs in
respect of goods shipped at Cochin port. It incurred expenses of ₹ 35 lakhs during the year
for operating its fleet. In respect of goods shipped at Rotterdam, Netherlands, it received ₹
50 lakhs in India. Its tax liability (rounded off) for the A.Y. 2021-22 is -
(A) ₹ 4,21,200
(B) ₹ 4,43,040
(C) ₹ 3,12,000
(D) ₹ 1,77,840

B;

Solution: The provisions of Section 44B would apply in this case. Thus, business income shall
be computed as follows –

Such freight shall form the part of gross receipts. Accordingly, gross receipts = ₹ 85 lakh + ₹
5 lakh + ₹ 2 lakh + ₹ 50 lakh = ₹ 142 lakh.

Therefore, profits and gains of business or profession = 7.5% of gross receipts i.e. 7.5% of ₹
142 lakh = ₹ 10,65,000.

Particulars ₹

Tax liability @ 40% of 10,65,000 4,26,000

Add: NEC @ 4% 17,040

Total tax payable 4,43,040

11. An applicant may withdraw an application for advance ruling within from the date
of the application.
(A) 15 days
(B) 45 days
(C) 30 days
(D) 60 days

C;

Solution: An applicant may withdraw an application within 30 days from the date of the
application.

12. Income from 'Salaries' which is payable by would be deemed to accrue or arise
in India:
(A) The Government of India to a citizen of India for services rendered outside India.
(B) The Government of India to a foreign citizen for services rendered outside India.
(C) The foreign Government to a non-citizen for services rendered outside India.
(D) The Government or any other person to a non-citizen or non-resident for services
rendered outside India.

A;

Solution: Income from 'Salaries' which is payable by the Government of India to a citizen of
India for services rendered outside India would be deemed to accrue or arise in India.

13. In case the assessee makes a false statement in any verification under equalization levy,
then he shall be punishable with imprisonment for a term which may extend to .
(A) 5 years and with fine
(B) 5 years or with fine or both
(C) 3 years and with fine
(D) 2 years and with fine
C;

Solution: In case the assessee makes a false statement in any verification under
equalization levy, then he shall be punishable with imprisonment for a term which may
extend to 3 years and with fine.

14. Any income by way of interest payable to a non-resident by a unit located in in


respect of monies borrowed by it on or after the 1st day of September, 2019 under section
10(15) (ix).
(A) a Special Economic Zone
(B) an International Financial Services Centre
(C) A Electronic hardware technology park
(D) Any of the above

B;

Solution: Any income by way of interest payable to a non-resident by a unit located in an


International Financial Services Centre in respect of monies borrowed by it on or after the
1st day of September, 2019.
Profit Gains Business

1. Which of the following export incentive is taxable under the head Profits and gains of
business and profession?
(A) Profit on sale of import entitlements:
(B) Cash assistance against exports under any scheme of Government of India
(C) Customs duty or excise re-paid or repayable as drawback
(D) All of the above

D;

Solution: As per provisions of Section 28(iiia) to (iiie), all of the above export incentives are
taxable under Profits and gains of business or profession.

2. Which of the following are included in business according to Section 2(13) (i) trade, (ii)
commerce (iii) manufacture (iv) any adventure or concern in the nature of trade, commerce
or manufacture.
(A) (i) and (ii)
(B) (i), (ii) and (iii)
(C) (i), (ii), (iii) and (iv)
(D) (i) and (iii)

C;

Solution: As per Section 2(13) 'Business' includes any trade, commerce or manufacture or
any adventure or concern in the nature of trade, commerce or manufacture.
3. The assessee has purchased stock for ₹ 1,50,000 on 12-07-2019. The same was converted
into capital asset on 10-07-2020.The fair market value of inventory on that date was ₹
1,80,000 and the same was sold on 15-03-2021 for ₹ 2,00,000. The reported business profits
for assessment year 2021-22.
(A) ₹ 30,000
(B) ₹ 50,000
(C) ₹ 20,000
(D) Nil

A;

Solution: Any profit or gains arising from conversion of inventory into capital asset or its
treatment as capital asset shall be charged to tax as business income. It is also provided that
the fair market value of the inventory on the date of conversion or treatment determined in
the prescribed manner, shall be deemed to be the full value of the consideration received
or accruing as a result of such conversion or treatment.

Thus, reported business profits will be [₹ 1,80,000 - ₹ 1,50,000] = ₹ 30,000.

4. Rate of depreciation chargeable on Motor buses, motor lorries and motor taxis used in a
business of running them on hire, acquired on or after 23-08-2019 but before 01-04-2020
and is put to use before the 01-04-2020 for the assessment year 2020-21 is —
(A) 45%
(B) 30%
(C) 15%
(D) 40%

A;
Solution: Rate of depreciation chargeable on fully Motor buses, motor lorries and motor
taxis used in a business of running them on hire, acquired on or after 23-08-2019 but before
01-04-2020 and is put to use before the 01-04-2020 is allowed @ 45%.

5. WDV of the block as on April 1, 2020 is ₹ 15,00,000; rate of depreciation = 15%. Assets,
falling within the block, are purchased on 1542-2020 for ₹ 5,00,000. Sale of entire block = ₹
8,80,000. Compute the amount of depreciation admissible.
(A) Nil
(B) ₹ 1,68,000
(C) ₹ 37,500
(D) ₹ 75,000

A;

Solution: Computation of depreciation admissible-

Particulars ₹

Opening WDV 15,00,000

Add: Actual cost of the asset acquired during the year 5,00,000

20,00,000

Less: Moneys payable in respect of entire block sold during the year 8,80,000

WDV as on 31-3-2021 (as block ceases to exist) -

Depreciation admissible -

Short term capital Loss (₹ 20,00,000 — ₹ 8,80,000) 11,20,000


6. Swan (Pvt.) Ltd. acquired machinery for ₹ 5,75,000 which included Goods and Services tax
of ₹ 75,000 eligible for Input tax credit. It borrowed ₹ 3,00,000 from a bank for purchase of
the said machine. Interest on the bank loan upto the date of usage of machine was
ascertained as ₹ 25,000. The machine was put to use from 15 th September, 2020. Assume
the rate of depreciation at 15%. The eligible amount of depreciation will be —
(A) ₹ 90,000
(B) ₹ 78,750
(C) ₹ 86,250
(D) ₹ 75,000

B;

Solution: Computation of depreciation admissible

Particulars ₹

Purchase price of machinery 5,75,000

Less: GST as ITC is admissible [Explanation 9 to section 43(1)] -75,000

Add: Interest on borrowed capital till the machine is put to use 25,000
[Explanation 8 to section 43(1)]

Actual Cost 5,25,000

Depreciation @ 15% 78,750

7. A company incurred capital expenditure on scientific research viz., (i) land ₹ 5 lakh; (ii)
building ₹ 10 lakh; and (iii) equipments ₹ 7 lakh. The amount of expenditure eligible for
deduction under section 35(i)(iv)would be —
(A) ₹ 22 lakh
(B) ₹ 17 lakh
(C) ₹ 15 lakh
(D) ₹ 5 lakh

B;

Solution: Any expenditure of a capital nature on scientific research related to the business
carried on by the assessee would be deductible in full in the previous year in which it is
incurred. No deduction will be allowed in respect of capital expenditure incurred on the
acquisition of any land whether the land is acquired as such or as part of any property.

Thus, amount to be allowed as deduction = ₹ 17 lakh.

8. GP Ltd. was incorporated on 31-12-2019 for manufacture of tyres and tubes for motor
vehicles. The manufacturing unit was set up on 30-4-2020. The company commenced its
operations on 1-5-2020. The total cost of the plant and machinery installed in the unit is ₹
100 crores. The said plant and machinery included second-hand plant and machinery
bought for ₹ 10 crores and new plant and machinery for scientific research relating to the
business of the assessee acquired at a cost of ₹ 10 crores. Compute the amount of normal
depreciation and additional depreciation allowable u/s 32 of the Income-tax Act, 1961 in
respect of the assessment year 2021-22.
(A) Normal Depreciation : ₹ 15 crores and ₹ 20 crores as additional depreciation
(B) Normal Depreciation : ₹ 13.5 crores and ₹ 20 crores as additional depreciation
(C) Normal Depreciation : ₹ 13.5 crores and ₹ 16 crores as additional depreciation
(D) Normal Depreciation : ₹ 13.5 crores and ₹ 18 crores as additional depreciation

C;

Solution: Computation of depreciation allowable for the A.Y. 2021-22 to GP Ltd.

Particulars ₹ in crores
Total cost of plant and machinery 100

Less: Used for Scientific Research 10

Cost eligible for normal depreciation 90

Normal depreciation at 15% of ₹ 90 crores 13.5

Additional Depreciation:

Cost of plant and machinery 100

Less: Second hand plant and machinery 10

Less: Plant and machinery used for scientific research 10

Cost eligible for additional depreciation 80

Additional Depreciation at 20% of actual cost of ₹ 80 crores (put to use 16


not less than 180 days)

9. The rate of depreciation admissible on oil well is:


(A) Nil
(B) 15%
(C) 25%
(D) 40%

B;

Solution: The rate of depreciation admissible on oil well is 15%.


10. What is the amount of deduction in relation to Tea, Coffee and Rubber development A/c,
u/s 33AB?
(A) 20% of profits of such business.
(B) 40% of profits of such business.
(C) 60% of profits of such business.
(D) 100% of profits of such business.

B;

Solution: A sum equal to the aggregate of the deposits made in development account or
40% of the profits of such business (as computed under the head 'Profits and gains of
business or profession' before making any deduction under this section), whichever is less
shall be eligible for deduction.

11. G Ltd. is engaged in the business of growing and manufacturing tea in India. For the
previous year ending on 31-03-2021 its composite business profits before allowing
deduction under section 33AB are ₹ 60,00,000. On 01-09-2020 it deposited a sum of ₹
11,00,000 in the Tea Development Account. During the previous year 2019-20 G Ltd., had
incurred a business loss of ₹ 14,00,000 which has been carried forward. The taxable income
under the head Profits and gains of business and profession for AY 2021-22 will be —
(A) ₹ 35,00,000
(B) ₹ 5,60,000
(C) (₹ 1,00,000)
(D) ₹ 40,000

B;

Solution: Computation of taxable business income —

Particulars ₹
Previous year 2020-21 :

Composite profits before allowing deduction u/s 33AB 60,00,000

Less: Deduction u/s 33AB, 40% of ₹ 60,00,000 or ₹ 11,00,000, whichever 11,00,000


is less

49,00,000

Less: 60% of T 49,001000 being agricultural income 29,40,000

Non-agricultural business income taxable u/s 28 19,60,000

Less: B/F Business loss of assessment year 2020-21 14,00,000

Taxable Business Income 5,60,000

12. A company engaged in the manufacturing of pharmaceutical products commenced its


business on 1-4-2020. During the financial years 2017-18 to 2019-20 it had incurred ₹ 2.00
lacs annually as expenditure on salaries and purchase of raw materials for the purpose of
research connected with its business. During the previous year 2020-21 it incurred on
scientific research, revenue expenditure of ₹ 2.00 lacs and a capital expenditure of ₹ 3.5 lacs
on purchase of plant and machinery. The amount of deduction admissible under Section 35
is —
(A) ₹ 11,50,000
(B) ₹ 14,25,000
(C) ₹ 17,25,000
(D) ₹ 8,25,000

A;

Solution: The deduction admissible under section 35 is as follows —


Particulars Section Qualifying % of weight Deduction
sum

Expenditure on salaries and purchase 35(1)(i) 6,00,000 100% 6,00,000


of raw materials incurred for
scientific research during period of 3
years before the commencement of
business i.e. during F.Y. 2017-18 to
2019-20 [T 2 lakhs x 3 years]

Revenue expenditure incurred for 35(2AB) 2,00,000 100% 2,00,000


P.Y. 2020-21

Capital expenditure on purchase of 35(2AB) 3,50,000 100% 3,50,000


plant and machinery

Total deduction u/s 35 11,50,000

Working Note: The assessee is a company engaged in the business of manufacture of


pharmaceutical products. Therefore, it is entitled to deduction u/s 35(2AB) to the extent of
100% in respect of current year expenditure on scientific research, in-house research and
development facility approved by the prescribed authority.

13. Which of the following payments are eligible for 100% weighted deduction in case of
company assessee
(i) any sum paid to a approved research association which has as its object the undertaking
of scientific research
(ii) any sum paid to a approved research association which has as its object the undertaking
of research in social science or statistical research
(iii) any expenditure on agricultural extension project notified by the CBDT
(iv) any expenditure (not being expenditure in the nature of cost of any land or building) on
any skill development project notified by the CBDT
(A) (i) and (iii)
(B) (i) and (iv)
(C) (i),(ii),(iii) and (iv)
(D) (ii) and (iv)

C;

Solution: The following payments are eligible for 100% weighted deduction in case of
company assessee—

(i) any sum paid to a approved research association which has as Its object the undertaking
of scientific research
(ii) any sum paid to a approved research association which has as Its object the undertaking
of research in social science or statistical research
(iii) any expenditure on agricultural extension project notified by the CBDT
(iv) any expenditure (not being expenditure in the nature of cost of any land or building) on
any skill development project notified by the CBDT.

14. If an individual assessee carries on any scientific research related to his business be shall be
allowed deduction of
(A) Revenue expenditure.
(B) Capital expenditure.
(C) Both revenue and capital expenditure except capital expenditure on acquisition of land.
(D) Both revenue and capital expenditure except capital expenditure on acquisition of land
and building.

C;
Solution: If an individual assessee carries on any scientific research related to his business
he shall be allowed deduction of both revenue and capital expenditure except capital
expenditure on acquisition of land.

15. Any capital expenditure incurred on acquiring telecom license is deductible in —


(A) 5 Equal installments
(B) 10 Equal installments
(C) 15 Equal installments
(D) Equally over the period of the license

D;

Solution: Any capital expenditure incurred on acquiring telecom license is deductible in


Equally over the period of the license as per the provisions of Section 35ABB.
Profit Gains Business Profession 2

1. Easy talk Ltd. has incurred capital expenditure amounting ₹ 108 crores on spectrum on 10-
07-2019. The period of spectrum is 10 years. The business commenced on 10-04-2020.
Determine the amount of deduction admissible under Section 35ABA for Assessment Year
2021-22.
(A) ₹ 10.8 crore
(B) ₹ 12 crore
(C) ₹ 108 crore
(D) Nil

B;

Solution: As per Section 35ABA, any capital expenditure incurred for acquiring any right to
operate spectrum for which payment has actually been made to obtain a spectrum, then,
there shall be allowed deduction during the period for which the spectrum shall remain
effective. Hence, the amount of deduction admissible shall be ₹ 108 crore ÷ 9 = ₹ 12 crore.

2. XYZ Limited purchased a machine on 1st April, 2020 for ₹ 10 crores by availing 70% loan
facility from bank. The machine was put to use into effective production on 1 st February,
2021. Interest on loan is charged at 12% per annum. The loan amount is outstanding on 31-
03-2021. Interest for the financial year ending 31-03-2021 is paid on 31-03-2021. Determine
admissible depreciation to the assessee.
(A) ₹ 3,50,00,000
(B) ₹ 1,75,00,000
(C) ₹ 1,50,00,000
(D) ₹ 1,87,25,000
D;

Solution: Section 36(1)(iii), has been amended to provide that interest paid in respect of
capital borrowed for acquisition of an asset, for the period upto the date on which the asset
is first put to use to be capitalized. Hence, interest for such period has to be capitalized, by
adding the same to the cost of the asset. Therefore, Interest @ 12% p.a. for a period of 10
months from 01-4-2020 to 31-1-2021 on ₹ 7 crores, being the amount of loan, has to be
capitalized.

Particulars ₹

Cost of machinery 10,00,00,000

Add: Interest [12% x 10/12 x ₹ 7,00,00,000] 70,00,000

Actual Cost of machinery 10,70,00,000

Interest @ 12% for two months (February 2021 & March 2021) after the 14,00,000
asset is put to use is allowable as deduction under section 36(1)(iii) [12% x
2/12 x ₹ 7,00,00,000]

Depreciation:

Particulars ₹

Since the machinery is put to use for less than 180 days in the P.Y. 2020-21, 80,25,000
the depreciation would be restricted to 50% of the amount calculated at
the prescribed percentage of 1S%.

Therefore, depredation = 50% x [15% x ₹ 10,70,00,000]

Likewise, the additional depreciation would also be restricted to 50% of the 1,07,00,000
amount calculated at the prescribed percentage of 20% assuming that XYZ
Ltd. is engaged in the manufacture or production of any article or thing and
that the machinery acquired is a new machinery.

Therefore, additional depredation = 50% x [20% x ₹ 10,70,00,000]

Total 1,87,25,000

3. Any asset in respect of which a deduction is claimed and allowed under Section 35AD shall
be used only for the specified business, for a period of beginning with the previous
year in which such asset is acquired or constructed.
(A) Five years
(B) Eight years
(C) Six years
(D) Three years

B;

Solution: Any asset in respect of which a deduction is claimed and allowed under Section
35AD shall be used only for the specified business, for a period of eight years beginning with
the previous year in which such asset is acquired or constructed.

4. XYZ Ltd. has incurred the following expenditure for the year ended 31-03-2021—

Particulars ₹

(i) Payment to approved association and institutions for carrying out rural 5,00,000
development programme

(ii) National Urban Poverty Eradication Fund 1,25,000


(iii) National Rural Development fund 1,00,000

After payment to approved association by XYZ lid, the approval granted to such association
was withdrawn by the prescribed authority. Determine the amount of deduction admissible
to XYZ Ltd. for the year ended 31-03-2021.

(A) ₹ 7,25,000
(B) ₹ 2,25,000
(C) ₹ 1,00,000
(D) ₹ 5,00,000

A;

Solution: Computation of admissible deduction —

Particulars ₹

(i) Payment to approved association and institutions for carrying out rural 5,00,000
development programme

(ii) National Urban Poverty Eradication Fund 1,25,000

(iii) National Rural Development fund 1,00,000

Total admissible deduction 7,25,000

5. What is the qualifying expenditure for deduction in relation to preliminary expenses u/s
35D:
(A) Aggregate amount of eligible expenditure
(B) 5% of cost of project
(C) Higher of (A) or (B)
(D) Lower of (A) or (B)
D;

Solution: Lower of the following qualifies for deduction —

a) Aggregate amount of eligible expenditure; or


b) Higher of (i) or (ii) —
(i) 5% of Cost of the project; or
(ii) 5% of Capital employed (Only in the case of an Indian company).

6. M/s. Raksha & Company paid ₹ 75,000 as customs duty and 84,000 as income tax during the
previous year. Calculate the amount of expenditure allowable:
(A) ₹ 1,59,000
(B) ₹ 84,000
(C) ₹ 75,000
(D) Nil

C;

Solution: Custom duty is allowed as deduction whereas income tax is not allowed as
deduction while computing profits and gains of business or and profession.

7. Mr. A took a loan of ₹ 10,00,000 on 15th June 2020 for acquisition of an asset and
immediately put the asset to use. Interest for the Previous Year 2020-21 on such loan
amounted to ₹ 65,000. Payment of ₹ 55,000 against the interest amount was made on 11th
May 2021 and balance amount was paid on 11th December 2021. The amount of revenue
expenditure allowable to assessee in Previous Year 2020-21 :
(A) Nil
(B) ₹ 65,000
(C) ₹ 55,000
(D) ₹ 10,00,000

C;

Solution: The entire amount of expenditure shall be regarded as revenue expenditure. Since
the payment of interest upto the due date of furnishing the return of income is ₹ 55,000,
the same shall be allowed as deduction in Financial Year 2020-21.

8. XYZ Ltd. was engaged in the business of hospitality services. It has purchased animals for
entertainment of guests amounting ₹ 5,00,000. ₹ 10,000 has been received on account of
sale proceeds of carcass of animal which died during the year. The amount deductible while
computing Profits and Gains of business and profession is —
(A) Nil
(B) 5,00,000
(C) 4,90,000
(D) 49,000

C;

Solution: Loss in respect of animals, used for business or profession (otherwise than as
stock-in trade), which have died or become permanently useless shall be allowed as
deduction. Deduction allowed = [Actual cost of the animal — Sale proceeds of carcasses or
animals]. Thus, amount to be allowed is ₹ 4,90,000.

9. As restructuring of its debt, XYZ Ltd. has converted arrears of interest of ₹ 3,00,000 on term
loan into a new term loan with a revised repayment schedule. The company has paid ₹
50,000 towards such funded interest during the year. A further sum of ₹ 50,000 paid before
due date of filing of return. The amount to be deductible while computing profits and gains
of business and profession is —
(A) ₹ 3,00,000
(B) ₹ 50,000
(C) ₹ 1,00,000
(D) Nil

C;

Solution: In case if any interest is payable on any loan or borrowing from any Public
Financial Institution or State Financial Corporation, or State Industrial Investment
Corporation; or on any loan or advances from a scheduled bank or a co-operative bank
other than a primary agricultural credit society or a primary co-operative agricultural and
rural development bank in accordance with the terms and conditions of the agreement
governing such loan or advance; and such interest is converted into loan or borrowing or
advance, then the same shall not be allowed as deduction in the year of conversion, but
shall be allowed in the year in which such converted loan is as actually paid. Thus, ₹ 50,000
shall be allowed as deduction. Besides this the amount of ₹ 50,000 which is paid before the
due date of filing of return of income will also be deductible. Hence, amount deductible = ₹
1,00,000.

10. An assessee was engaged in the business of dealing in commodities. He had paid
Commodities transaction tax of ₹ 75,000 in respect of the taxable commodities
transactions. Income arising of ₹ 3,00,000 from such taxable commodities transactions was
included in the income computed under the head "Profits and gains of business or
profession". Such expenditure of payment of Commodities transaction tax :
(A) Shall be disallowed
(B) Shall be allowed as revenue expenditure
(C) Shall be allowed in 5 equal instlaments
(D) Shall be allowed in 10 equal instalments

B;
Solution: Section 36(1)(xvi) provides that an amount equal to the UT paid by the assessee in
respect of the taxable commodities transactions entered into in the course of his business
during the previous year shall be allowable as deduction, if the income arising from such
taxable commodities transactions is included in the income computed under the head
"Profits and gains of business or profession". Thus, ₹ 75,000 shall be allowed as deduction.

11. An assessee paid ₹ 7,50,000 as goods and services tax, ₹ 5,00,000 as income tax and ₹
50,000 as tax on non-monetary perquisites of employees. The amount to be disallowed
while computing profits and gains of business and profession is —
(A) ₹ 7,50,000
(B) ₹ 5,00,000
(C) ₹ 50,000
(D) ₹ 5,50,000

D;

Solution: Income tax (₹ 5,00,000) and tax on non-monetary perquisites (₹ 50,000) shall not
be allowed while computing profits and gains of business and profession. Hence, amount
not deductible = ₹ 5,50,000.

12. Delta Ltd. credited the following amounts to the account of resident payees in the month of
March, 2021 without deduction of tax at source. What would be the amount of
disallowance on account of non-deduction of tax at source by Delta Ltd. on these amounts
during the financial year 2020-21, assuming that tax has been deducted at source and paid
in April 2021 at the time of making subsequent payments?

Particulars ₹

(i) Salary to its employees (credited and paid in March, 2021) 12,00,000
Directors' remuneration (credited in March, 2021 and paid in April, 2021) 28,000
(ii)

(A) ₹ 12,28,000
(B) ₹ 12,00,000
(C) ₹ 3,68,400
(D) Nil

C;

Solution: Non-deduction of tax at source on any sum payable to a resident on which tax is
deductible at source as per the provisions of Chapter XVII-B would attract disallowance
under section 40(a)(ia).

Therefore, non-deduction of tax at source on any sum paid by way of salary on which tax is
deductible under section 192 or any sum credited or paid by way of directors' remuneration
on which tax is deductible under section 194J, would attract disallowance @ 30% under
section 40(a)(ia).

Whereas in case of salary, tax has to be deducted under section 192 at the time of payment,
in case of directors' remuneration, tax has to be deducted at the time of credit of such sum
to the account of the payee or at the time of payment, whichever is earlier.

Therefore, in both the cases i.e., salary and directors' remuneration, tax is deductible in the
P.Y.2020-21, since salary was paid in that year and directors' remuneration was credited in
that year. Therefore, the amount to be disallowed u/s 40(a)(ia) while computing business
income for A.Y. 2021-22 is as follows —

Particulars ₹

(i) Salary to its employees (credited and paid in March, 2021) 3,60,000
(ii) Directors' remuneration (credited in March, 2021 and paid in April, 2021) 8,400

Total amount disallowed 3,68,400

13. Varun Ltd. paid fees for technical services of ₹ 6 lakh to Ramana Ltd. (an Indian Company)
but omitted to deduct tax at source and such omission continued till the ‘due date' for filing
the return of income specified in section 139(1). The amount of expenditure liable for
disallowance would be —
(A) ₹ 1,80,000
(B) ₹ 6,00,000
(C) ₹ 1,20,000
(D) Nil

A;

Solution: Non-deduction of tax at source on any sum payable to a resident on which tax is
deductible at source as per the provisions of Chapter XVII-B would attract disallowance @
30% under section 40(a)(ia).

Hence, the amount to be disallowed shall be 30% of ₹ 6,00,000 = ₹ 1,80,000.

14. Ravi & Co. paid ₹ 40,000 by cash to Mr. Balu a supplier on 5-9-2020. The cash payment was
made on the day on which the bank was on strike. The amount of expenditure liable for
disallowance under section 40A(3) is :
(A) ₹ 40,000
(B) ₹ 12,000
(C) ₹ 30,000
(D) Nil
A,

Solution: Now payment made on day when the banks are closed is not covered in Rule 6DD,
hence the same shall be disallowed.

15. Ashish made cash payment of % 11,000 on 28 th March, 2021 in respect of purchases of
goods. Calculate the amount of expenditure to be disallowed u/s 40A(3).
(A) NIL
(B) ₹ 11,000
(C) ₹ 15,000
(D) ₹ 20,000

B;

Solution: Whole of the payment so made in cash shall be disallowed under Section 40A(3)
i.e. = ₹ 11,000.
Residential Status

1. Amongst the following which is the basic condition to identify the residential status of the
individual —
(A) Person must be in India for or more 182 days during the previous year.
(B) Person must be in India for less than 60 days during the previous year and 365 days on
more during the four years preceding the previous year.
(C) He must be in India in nine out of ten previous years preceding that year.
(D) He is a citizen of India, or a person of Indian origin, having total income, other than the
income from foreign sources, exceeding ₹ 15 lakh during the previous year, and he
comes to India for the purpose of visit to India during the relevant previous year and he
has been in India for a period or periods amounting in all to 120 days or more but less
than 182 days.

A;

Solution: Basic conditions as per Section 6(1) is : (1) He must be in India for a period of 182
days or more during the previous year , or (2) He must be in India for a period of 60 days or
more during the previous year and 365 days or more during the four years preceding the
previous year.

2. X, an Indian citizen, who is living in Delhi since 1981, left for Japan on 1st July, 2019 for
employment. He came back to India on 1st January, 2021 on a visit and stayed for 4 months.
His residential status for the assessment year 2021-22 would be —
(A) Resident and ordinarily resident
(B) Not ordinarily resident
(C) Non resident
(D) Resident.
C;

Solution: In case of Indian citizen who comes to India for the purpose of visit to India, basic
condition no 1 is applicable for being resident in India. In this case he is non-resident in
India, since his period of stay in India in P.Y. 2020-21 is for 4 months.

3. Which of the following conditions are to be satisfied for an individual shall be deemed to be
resident in India under Section 6(1A) of the Income-tax Act, 1961.
(A) He must be citizen of India;
(B) His total income, other than the income from foreign sources, must exceed ₹ 15 lakh
during the previous year
(C) He is not liable to tax in any other country or territory by reason of his domicile or
residence or any other criteria of similar nature.
(D) All of the above

D;

Solution: An individual shall be deemed to be resident in India if he fulfills the following


conditions:

(1) He must be citizen of India;


(2) His total income, other than the income from foreign sources, must exceed ₹ 15 lakh
during the previous year; and
(3) He is not liable to tax in any other country or territory by reason of his domicile or
residence or any other criteria of similar nature.

4. The residential status of HUF depends upon the .


(A) Control and management of affairs of business.
(B) Place of residence of Karta.
(C) Place of residence of Karta its members.
(D) Location of the head office.

A;

Solution: The residential status of HUF depends upon the Control and management of
affairs of business.

5. Find the residential status of HUF whose control and management is partly in India and the
Karta stays in India for 750 days during the period of 7 years proceeding the previous year
and is resident in last 4 years proceeding the previous year.
(A) Resident
(B) Non-Resident
(C) Not Ordinarily Resident
(D) Any of these

A;

Solution: According to Section 6(2), HUF is resident in India if the control and management
of its affairs is situated wholly or partly in India.

A resident HUF becomes not-ordinarily resident, if karta or manager satisfies one or both of
the following additional conditions :

 He has been non-resident in India in 9 out of the 10 years immediately preceding the
relevant previous year; or
 He has been in India for 729 days or less during the 7 years immediately preceding the
previous year.

Hence, HUF is said to be resident In India.


6. Somesh was born in New Jersey in 1977 and his father was born in China in 1949 whereas
Somesh's grandfather was born in India in 1933. Somesh visited India for 180 days during
the previous year 2020-21. He does not have any income in India. His residential status is —
(A) Resident
(B) Not Ordinarily Resident
(C) Not Resident
(D) Any of these

C;

Solution: Somesh is person of Indian origin. In case of person of Indian origin who comes to
India for the purpose of visit to India, basic condition no 1 is applicable. In this case he is
non-resident in India, since his period of stay in India in P.Y. 2020-21 is 180 days only.

7. Which of the following income is not taxable in case of non-resident


(A) Capital gains on transfer of land situated in India received in India
(B) Capital gains on transfer of land situated outside India received in India
(C) Capital gains on transfer of land situated outside India received outside India
(D) Capital gains on transfer of land situated in India received outside India

C;

Solution: The income which accrues and arises outside India and received outside India is
not taxable in India in hands of non-resident.

8. Determine the taxability of income of US based company Ralli Ltd., in India on entering
following transactions during the financial year 2020-21 :
(i) ₹ 4 lacs from a non-resident for use of patent for a business in India.
(ii) ₹ 8 lacs from a non-resident Indian for use of know- how for a business in Singapore.
(A) ₹ 4,00,000
(B) ₹ 8,00,000
(C) ₹ 12,00,000
(D) Nil

A;

Solution: A non-resident is chargeable to tax in India in respect of following incomes:

(i) Income received or deemed to be received in India.

(ii) Income accruing or arising or deemed to accrue or arise in India. In view of the above
provisions, taxability of income is determined in following manner:

Particulars ₹

1. The income from business set up in India is taxable in India. Therefore, 4,00,000
money received from a non- resident for use of patent for a business In India
is taxable in India.

2. Money received from a non-resident India for use of know-how for a Nil
business in Singapore is for the business outside India, therefore non-taxable
in India.

Total Income taxable in India 4,00,000

9. Income chargeable under the head "Salaries" payable by the Government to a citizen of
India for service rendered outside India is taxable if the person is —
(A) Resident
(B) Not Ordinarily Resident
(C) Not Resident
(D) Any of these
D;

Solution: Income chargeable under the head "Salaries" payable by the Government to a
citizen of India for service rendered outside India is deemed to accrue or arise in India,
hence it is taxable whether the individual is resident, NOR or NR in India.

10. Following are the particulars of Income of Adarsh –

Particulars ₹

(i) Income from agriculture in Indonesia being invested there only 12,350

(ii) Income from business in Bangladesh being controlled from India 10,150

If Adarsh is NOR what is his total income?

(A) ₹ 12,350
(B) ₹ 10,150
(C) ₹ 22,500
(D) ₹ 2,200

B;

Solution: The Total Income in case of NOR is as under —

Particulars ₹

(i) Income from agriculture in Indonesia being invested there only -

(ii) Income from business in Bangladesh being controlled from India 10,150

Total Income 10,150


11. Income of non-resident when attributed from operations in India relating to the following is
taxable in India : (1) Profits of business (2) Fee for technical services (3) Royalty (4) Interest
Select the correct answer from the options given below —
(A) (1)
(B) (1), (3) and (4)
(C) (1) and (3)
(D) (1), (2), (3) and (4).

A;

Solution: The concept of attribution is applicable in case of profits of business only.

12. From the following particulars of Income furnished by Mr. K pertaining to the year ended
31-03-2021, -

Particulars ₹

(i) Interest received from Indian Government bonds outside India 15,000

(ii) Royalty received outside India from non-resident who used the rights for 10,000
business outside India

(iii) Agricultural income from lands in Japan (20% Is received In India) 37,000

If Mr. K is non-resident what is his total income?

(A) ₹ 15,000
(B) ₹ 20,000
(C) ₹ 7,400
(D) ₹ 22,400

D;
Solution: The total income of non- resident is as under—

Particulars ₹

(i) Interest received from Indian Government bonds outside India 15,000

(ii) Royalty received outside India from non-resident who used the rights for -
business outside India [Shall not be deemed to accrue or arise In India]

(iii) Agricultural income from lands in Japan (20% Is received In India) 7,400

Total Income 22,400

13. Dividends received from an Indian company amounting ₹ 1,00,000 in USA is in case
of a resident, in case of a not ordinarily resident and in case of a
non-resident?
(A) Taxable, exempt, exempt
(B) Taxable, taxable, taxable
(C) Taxable, taxable, exempt
(D) Exempt, exempt, exempt

D;

Solution: Dividends received from an Indian company shall be deemed to accrue or arise in
India, hence taxable. However, dividend upto ₹ 10,00,000 is exempt from tax.

14. From the following particulars of Income furnished by Mr. x resident in India pertaining to
the year ended 31-03-2021,–
Particulars ₹

(i) Salary (computed) received from US based employer for services rendered 75,000
in India

(ii) Income from property in US received in US 37,000

(iii) Agricultural Income received in US from lands situated in India 1,00,000

What is his total income?

(A) ₹ 75,000
(B) ₹ 37,000
(C) ₹ 1,12,000
(D) ₹ 2,12,000

C;

Solution: Total Income is as under—

Particulars ₹

(i) Salary (computed) received from US based employer for services rendered 75,000
in India

(ii) Income from property in US received in US 37,000

(iii) Agricultural Income received in US from lands situated in India Exempt

Total Income 1,12,000

15. From the following particulars of Income furnished by Ms. G non-resident in India
pertaining to the year ended 31-03-2021,—
Particulars ₹

(i) Interest on debentures in an Indian company (Ms. G received the same in 18,500
Malaysia)

(ii) Short-term capital gain on sale of shares of an Indian company 15,000

(iii) Interest on fixed deposit receipt with SBI 12,000

What is her total income?

(A) ₹ 45,500
(B) ₹ 27,000
(C) ₹ 33,500
(D) ₹ 30,500

A;

Solution: The total income of NR shall be determined as under—

Particulars ₹

(i) Interest on debentures in an Indian company [Income is deemed to be 18,500


accrue in India)

(ii) Short-term capital gain on sale of shares of an Indian company [Income is 15,000
deemed to be accrue in India]

(iii) Interest on fixed deposit receipt with SBI [Income is deemed to be accrue in 12,000
India]

Total Income 45,500


Salaries

1. Mr. A joins a job on 1-7-2020 at monthly salary of ₹ 25,000. His salary becomes due on last
day of each month. His taxable salary for AY 2021-22 will be —
(A) ₹ 1,85,000
(B) ₹ 1,75,000
(C) ₹ 2,50,000
(D) ₹ 2,25,000

B;

Solution: The income from salary shall be as under—

Particulars ₹

Gross Salary (₹ 25,000 x 9) 2,25,000

Less: Standard deduction u/s 16(ia) 50,000

Income under the head Salaries 1,75,000

2. Arun, a resident of Meerut, receives 38,000 per annum as basic salary. In addition, he gets ₹
12,000 p.a. as dearness allowance, which does not form part of retirement benifits, 5%
commission on turnover achieved by him (turnover achieved by him during the relevant
previous year is ₹ 6,00,000) and ₹ 7,000 per annum as house rent allowance. He, however,
pays ₹ 8,000 per annum as house rent. The quantum of house rent allowance exempt from
tax is—
(A) Nil
(B) ₹ 8,000
(C) ₹ 7,000
(D) ₹ 1,200

D; Solution: HRA is exempt to the extent least of the following —

Particulars ₹

Actual HRA received 7,000

Rent paid - 10% of salary [8,000 – 10% of (₹ 38,000 + 30,000)] 1,200

40% of the salary [40% of ₹(38,000 + 30,000)] 27,200

Exempted HRA 1,200

3. Naveen an employee of XYZ Ltd. receives ₹ 30,000 p.m. as basic salary. In addition, he gets ₹
10,000 p.m. as dearness allowance, not forming part of salary and ₹ 2,000 p.m as fixed
medical allowance. His gross salary is —
(A) ₹ 5,04,000
(B) ₹ 4,89,000
(C) ₹ 4,80,000
(D) ₹ 3,84,000

A;

Solution:

Particulars ₹

Basic Salary (₹ 30,000 x 12) 3,60,000

Dearness Allowance (₹ 10,000 x 12) 1,20,000


Fixed Medical allowance (₹ 2,000 x 12) 24,000

Gross Salary 5,04,000

4. Credit card facility provided to employee by the employer is —


(A) Exempt from tax.
(B) Taxable in case of specified employees only.
(C) Taxable in case of non-specified employees only.
(D) Taxable whether the employee is a specified employee or non-specified employee.

D;

Solution: Credit card facility provided to employee by the employer is taxable whether the
employee is a specified employee or non-specified employee.

5. Chhavi receives ₹ 25,000 p.m. as basic salary from ABC Ltd. She is also provided dearness
allowance of ₹ 15,000 p.m. forming pan of retirement benefit and has been provided rent
free accommodation, which is taken on rent, in Delhi (population exceeds 25 lakhs). The
employer has paid monthly rent of ₹ 10,000. The taxable value of rent-free accommodation
will be —
(A) ₹ 36,000
(B) ₹ 48,000
(C) ₹ 72,000
(D) ₹ 1,20,000

C;

Solution: The value of rent-free accommodation is Actual amount of lease rental paid or
payable by the employer or 15% of salary whichever is lower.
Particulars ₹

Basic Salary [₹25,000 x 12[ 3,00,000

Dearness Allowance it [₹ 15,000 x 12] 1,80,000

Salary for accommodation 4,80,000

15% of salary [A] 72,000

Lease rent paid by the employer [B] 1,20,000

Value of accommodation [A] or [B] whichever is lower 72,000

6. Garima receives ₹ 25,000 p.m. as bask salary from ABC Ltd. She is also provided dearness
allowance of ₹ 15,000 p.m. forming part of retirement benefit. She has been provided
accommodation in hotel in Delhi (population exceeds 25 lakhs). The employer has paid
monthly rent of ₹ 8,000 for such hotel accommodation. The taxable value of
accommodation for NJ 2021-22 will be —
(A) ₹ 1,15,200
(B) ₹ 96,000
(C) ₹ 72,000
(D) ₹ 48,000

B;

Solution: The value of rent-free accommodation in hotel is 24% of salary or lease rent paid
by the employer whichever is less —

Particulars ₹

Basic Salary (₹ 25,000 x 12) 3,00,000


Dearness Allowance (₹ 15,000 x 12) 1,80,000

Salary for accommodation 4,80,000

24% of salary [A] 1,15,200

Lease rent paid by the employer [B] 96,000

Value of accommodation [A] or [B] whichever is lower 96,000

7. Ms. R receives ₹ 50,000 p.m. as basic salary from ABC Ltd. She is also provided dearness
allowance of ₹ 25,000 p.m. forming part of retirement benefit. She has been provided
motor car (engine capacity exceeds 1.6 Its) owned by employer for her personal use. The
original cost of car is ₹ 6,00,000 (WDV ₹ 5,10,000). Expenditure incurred by the employer on
running and maintenance of the motor car during the relevant previous year amounted ₹
75,000. The salary of driver paid by the employer —₹ 96,000. The taxable value of car
facility for AY 2021-22 will be —
(A) ₹ 2,31,000
(B) ₹ 39,600
(C) ₹ 2,22,000
(D) ₹ 1,71,000

A;

Solution: The taxable value of car facility when used for personal purpose.

Particulars ₹

Depreciation of car @ 10% p.a. of original cost of car 60,000

Running and maintenance expenses 75,000


Salary for driver 96,000

Taxable Value of car facility 2,31,000

8. Jasveer receives ₹ 30,000 p.m. as basic salary from ABC Ltd. He is also provided dearness
allowance of ₹ 15,000 p.m. forming part of retirement benefit. He has been provided cook
for which the actual cost to employer is ₹ 60,000. Jasveer has paid ₹ 20,000 to the cook. The
taxable value of cook perquisite will be —
(A) ₹ 60,000
(B) ₹ 80,000
(C) ₹ 40,000
(D) Nil

A;

Solution: Actual expenditure incurred by the employer will be taxable value of perquisite.
Amount paid by Jasveer to cook will not be adjusted since the same is not paid to the
employer. Hence, taxable value of cook facility will be ₹ 60,000.

9. During the previous year 2020-21, Barun received a watch worth Z 20,000 from his
employer. The taxable value of the watch will be —
(A) ₹ 15,000
(B) ₹ 20,000
(C) Nil
(D) None of the above.

B;
Solution: The value of any gift, or voucher, or token in lieu of which such gift may be
received by the employee or by member of his household on ceremonial occasions or
otherwise from the employer shall be determined as the sum equal to the amount of such
gift. If the value exceeds ₹ 5,000, entire value will be chargeable to tax.

10. Himalaya Ltd. reimburses the following expenditure on medical treatment of the son of an
employee Karan. The treatment was done at UK :
(i) Travelling expenses ₹ 1,15,000.
(ii) Stay expenses at UK permitted by RBI ₹ 45,000 (Actual expenses ₹ 70,000).
(iii) Medical expenses permitted by RBI 50,000 (Actual expenses ? 70,000).

The taxable perquisites in the hands of Karan, if his annual income from salary was ₹
1,56,000 before including above expenditures will be—

(A) 2,55,000
(B) 45,000
(C) 1,60,000
(D) Nil

C;

Solution: In case salary is ₹ 1,56,000, taxable value of medical perquisite will be—

Particulars ₹

Stay expenses [₹ 70,000 — ₹ 45,000 (exempt)] 25,000

Medical expenses [₹ 70,000 — ₹ 50,000 (exempt)] 20,000

Travelling expenses (See Note) 1,15,000

Taxable value of Medical perquisite 1,60,000


Note : Cost of travel of employee or his family member and one attendant who
accompanies the patient in connection with such treatment is exempt only if gross total
income of such employee, as computed before including cost of travel, does not exceed ₹ 2
lakh.

11. A Ltd. has advanced an Interest free loan of ₹ 5,00,000 to B for purchase of car on 1-5-2020.
B has been repaying the loan in Installments of ₹ 20,000 p.m. on the 1 st of next month.
Compute the value of perquisite on account of interest assuming the Interest charged by
SBI is 10% p.a.
(A) ₹ 34,833
(B) ₹ 36,667
(C) ₹ 40,000
(D) ₹ 50,000

B;

Solution: The taxable value of perquisite will be arrived as under—

Month Maximum outstanding monthly balance Perquisite value


(amount in ₹)

April 2020 Nil Nil

May 2020 5,00,000 4,167

June, 2020 4,80,000 4,000

July, 2020 4,60,000 3,833

August, 2020 4,40,000 3,667

September, 2020 4,20,000 3,500


October, 2020 4,00,000 3,333

November, 2020 3,80,000 3,167

December, 2020 3,60,000 3,000

January, 2021 3,40,000 2,833

February, 2021 3,20,000 2,667

March, 2021 3,00,000 2,500

Total 36,667

12. Gyanesh is employee of ABC Pvt. Ltd. He receives ₹ 30,000 p.m. as basic salary. He is also
provided dearness allowance of ₹ 15,000 p.m. forming part of retirement benefit. The
employer has paid credit card fees of ₹ 15,000 and also amount of ₹ 35,000 on account of
credit card bills of purchases made by Gyanesh for his personal purposes. ₹ 5,500 is
recovered from Gyanesh by the employer. The taxable value of perquisite will be —
(A) ₹ 44,500
(B) ₹ 50,000
(C) ₹ 29,500
(D) Nil

A;

Solution: The amount of expenses including membership fees and annual fees incurred by
the employee or any member of his household, which is charged to a credit card (including
any add-on-card) provided by the employer, or otherwise, paid for or reimbursed by such
employer shall be taken to be the value of perquisite chargeable to tax as reduced by the
amount, if any paid or recovered from the employee for such benefit or amenity. Thus,
taxable value = ₹ 15,000 + ₹ 35,000 – ₹ 5,500 = ₹ 44,500.

13. Mr. Sunil Is a private sector employee. He received Z 12,00,000 as gratuity. He retired on
16th February 2021 after rendering 25 years and 7 months of service. His basic salary was Z
30,000 p.m. His dearness allowance was 15,000 p.m. (40% forms part of retirement
benefit). He is not covered under Payment of Gratuity Act, 1972. The amount of gratuity
exempt will be —
(A) ₹ 5,40,000
(B) ₹ 10,00,000
(C) ₹ 4,50,000
(D) ₹ 12,00,000

C;

Solution: The amount of exemption shall be —

Particulars ₹ ₹

Gratuity received 12,00,000

Exemption u/s 10(10) : Least of the following is exempt –

Gratuity received 12,00,000

1/2 x Average Salary (₹ 36,000) x Completed Years (25) 4,50,000

Specified limit 20,00,000 4,50,000

Taxable Gratuity = Gratuity Received — Exemption 7,50,000


14. Mr. Jain retires from a Private service on 1st August 2020. At the time of his retirement after
28 years and 8 months of service, he was getting the salary of ₹ 34,000 per month. He gets
pension of ₹10,000 per month upto 31st December 2020. With effect from 1st January 2021
he gets 4/5th of his pension commuted for ₹ 12,00,000. Determine his gross salary taxable
for AY 2021-22 will be —
(A) ₹ 6,42,000
(B) ₹ 6,66,000
(C) ₹ S,86,000
(D) ₹ 1,92,000

A;

Solution: Computation of gross salary —

Particulars ₹

Salary @ ₹ 34,000 per month (April 2020 to July 2020) 1,36,000

Uncommuted pension [₹ 10,000 x 5 months (August 2020 to December 2020) + 56,000


[₹ 2,000 x 3 (January 2020 to March 2020)]

[Uncommitted Pension: Since Mr. Join commuted 4/5 th pension in December


2020. His uncommitted pension from January 2021 onward shall be [₹10,000 ×
1/5) = ₹ 2,000 p.m.]

Taxable portion of commuted value of pension 4,50,000

[Commuted pension: % of pension commuted = 4/5 = 80%.

Commuted value of pension (Pension received ÷ % commuted) = ₹ 12,00,000 ÷


80% = ₹ 15,00,000.

Exemption = 1/2 of commuted value of pension (₹ 15,00,000) = ₹ 7,50,000.


Taxable portion of commuted pension = ₹ 12,00,000 — ₹ 7,50,000 = ₹ 4,50,000]

Gross salary 6,42,000

15. Mr. A (65 years) submits the following information for the Assessment year 2021- 22 :
 Gross salary — ₹ 8,80,000
 Income from other sources — ₹ 60,000
 Contribution to PPF — ₹ 70,000
 Compute the tax liability of A.
(A) ₹ 74,000
(B) ₹ 79,040
(C) ₹ 76,960
(D) ₹ 86,520

C;

Solution:

Particulars ₹

Gross salary 8,80,000

Less: Standard deduction u/s 16(ia) 50,000

Income from salary 8,30,000

Income from Other Sources 60,000

Gross Total Income 8,90,000

Less: Section 80C 70,000


Taxable Income 8,20,000

Tax liability 74,000

Add: HEC @ 4% 2,960

Total Tax 76,960


Settlements Appeal

1. The order passed by the Assessing Officer when challenged before the Commissioner
(Appeals) under section 246A, memorandum of appeal should be filed in —
(A) Form No. 35
(B) Form No. 36
(C) Form No. 36A
(D) Form No. 38

A;

Solution: The order passed by the Assessing Officer when challenged before the
Commissioner (Appeals) under section 246A, memorandum of appeal should be filed in
Form No. 35.

2. Mr. Balwant received assessment order passed under section 143(3) on 10-1-2020. He
wants to prefer an appeal before CIT (Appeals) against the assessment order. The time limit
for preferring appeal is days from the date of service of notice of demand.
(A) 15
(B) 30
(C) 35
(D) 60

B;

Solution: An appeal to the Commissioner (Appeals) against any assessment order which is
appealable is to be presented within 30 days from the date of service of notice of demand.
3. The time-limit for making revisional order under section 263(2) and 263(3) is -
(A) 6 Months from the date of assessment
(B) 6 Months from the date of order
(C) One year from the end of the financial year in which the order was passed
(D) None of the above

D;

Solution: No order under Section 263 shall be made after the expiry of 2 years from the end
of the financial year in which the order sought to be revised was passed.

4. Penalty under Section 271DA shall be imposed by


(A) Assistant Commissioner
(B) Joint Commissioner
(C) Additional Commissioner
(D) Commissioner

B;

Solution: Penalty under Section 271DA shall be imposed by Joint Commissioner.

5. Permissive tax planning is -


(A) Objective of tax planning
(B) Area of tax planning
(C) Type of tax planning
(D) None of the above

C;

Solution: Permissive tax planning is a type of tax planning.


6. XYZ Ltd. has furnished a return with income of 25,000. The assessment was completed
under Section 143(3) in which the total income computed by the assessing officer
amounted ₹ 75,000. XYZ Ltd. approached its consultant and asked him to file an appeal
before the Commissioner of Income-tax (Appeals) against the said assessment order.
Determine the appeal fee that is required to be paid by XYZ Ltd. for filing the said appeal.
(A) ₹ 500
(B) ₹ 250
(C) ₹ 1,000
(D) ₹ 750

B;

Solution: Where the total income of the assessee as computed by the Assessing Officer
does not exceed ₹ 1,00,000, it is to be accompanied by fees of ₹ 250.

7. In every appeal, the Commissioner (Appeals), where it is possible, may hear and decide such
appeal within a period of
(A) 1 year from date on which such appeal is filed before him
(B) 1 year from the end of the financial year in which such appeal is filed before him
(C) 6 months from the end of the financial year in which such appeal is filed before him
(D) 6 months from the date on which such appeal is filed before him.

B;

Solution: In every appeal, the Commissioner (Appeals), where It is possible, may hear and
decide such appeal within a period of 1 year from the end of the financial year in which such
appeal is filed before him.
8. The Appellate Tribunal may, at any time within which order is passed, with a view
to rectifying any mistake apparent from records, amend any order passed by it.
(A) 6 months from the end of the month in
(B) 6 months from the date on
(C) 4 years from the end of financial year in
(D) 4 years from the date on

A;

Solution: The Appellate Tribunal may, at any time within 6 months from the end of the
month in which order is passed, with a view to rectifying any mistake apparent from
records, amend any order passed by it.

9. The maximum period (including extension) for which stay granted by the Appellate Tribunal
shall remain in force is .
(A) 180 days
(B) 365 days
(C) 90 days
(D) Indefinite time

B;

Solution: The maximum period (including extension) for which stay granted by the
Appellate Tribunal shall remain in force is 365 days.

10. Every settlement application should be accompanied by the prescribed fees of .


(A) ₹ 500
(B) ₹ 1,500
(C) ₹ 1,000
(D) ₹ 10,000

A;

Solution: Every settlement application should be accompanied by the prescribed fees of ₹


500.

11. The Settlement Commission shall pass final order within


(A) 12 months from the end of the month in which the application was made.
(B) 18 months from the end of the month in which the application was made.
(C) 6 months from the end of the month in which the application was made.
(D) 24 months from the end of the month in which the application was made

B;

Solution: The Settlement Commission shall pass final order within 18 months from the end
of the month in which the application was made.

12. If any person refuses to sign any statement made by him in the course of any proceedings
under this Act, which an income-tax authority may legally require him to sign, he shall pay,
by way of penalty, a sum of .
(A) ₹ 10,000
(B) ₹ 25,000
(C) ₹ 50,000
(D) ₹ 1,00,000

A;
Solution: If any person refuses to sign any statement made by him in the course of any
proceedings under this Act, which an income-tax authority may legally require him to sign,
he shall pay, by way of penalty, a sum of ₹ 10,000.

13. If any person fails to comply with a notice issued under Section 143(2), he shall pay, by way
of penalty, a sum of .
(A) ₹ 1,000
(B) ₹ 10,000
(C) ₹ 50,000
(D) ₹ 1,00,000

B;

Solution: If any person fails to comply with a notice issued under Section 143(2), he shall
pay, by way of penalty, a sum of ₹ 10,000.

14. Whoever fraudulently removes, conceals, transfers or delivers to any person, any property
or any interest therein, intending thereby to prevent that property or interest therein from
being taken in execution of a certificate under the provisions of the Second Schedule shall
be punishable with rigorous imprisonment for a term which may extend to
(A) two years and shall also be liable to fine.
(B) seven years and shall also be liable to fine.
(C) 6 months and shall also be liable to fine.
(D) three years and shall also be liable to fine.

A;

Solution: Whoever fraudulently removes, conceals, transfers or delivers to any person, any
property or any interest therein, intending thereby to prevent that property or interest
therein from being taken in execution of a certificate under the provisions of the Second
Schedule shall be punishable with rigorous imprisonment for a term which may extend to
two years and shall also be liable to fine.

15. If a person wilfully attempts in any manner whatsoever to evade any tax, penalty or interest
chargeable or imposable, or under reports his income, under this Act, he shall be
punishable, in a case where the amount sought to be evaded or tax on under-reported
income do not exceed 25,00,000 with rigorous imprisonment for a term which shall not be
less than but which may extend to .
(A) Three months; seven years and with fine.
(B) Six months; seven years and with fine.
(C) Three months; two years and with fine.
(D) Six months; two years and with fine.

C;

Solution: If a person wilfully attempts in any manner whatsoever to evade any tax, penalty
or interest chargeable or imposable, or under reports his income, under the Act, he shall be
punishable, in a case where the amount sought to be evaded or tax on under-reported
income do not exceed 25,00,000 with rigorous imprisonment for a term which shall not be
less than three months but which may extend to two years and with fine.
TDS TCS Advance Tax Recovery

1. X Ltd. pays salary of ₹ 8,00,000 (computed) to Mr. Kamlesh. Mr. Kamlesh has reported a loss
from house property of ₹ 2,00,000 to his employer. The monthly amount of tax deducted at
source will be —
(A) ₹ 2,817
(B) ₹ 6283
(C) ₹ 1,733
(D) Nil

A;

Solution: Computation of TDS liability —

Particulars ₹

Income under the head Salaries 8,00,000

Less: Loss from house property 2,00,000

Total Income 6,00,000

Tax on Total Income including HEC 33,800

TDS liability [Monthly deduction] [t 33,800 + 12] 2,817

2. The amount of TDS payable on the sum of ₹ 25,000 payable to X Ltd. By Government of
India by way of interest on securities owned by it would be —
(A) ₹ 2,500
(B) ₹ 75,150
(C) Nil
(D) ₹ 2,600

C;

Solution: No tax shall be deducted under Section 193 from any interest payable on any
security of the Central Government or a State Government. However, tax shall be deducted
@ 7.5% if the interest payable on 7.75% Savings (Taxable) Bonds, 2018 exceeds ₹ 10,000
during the financial year.

3. Tax shall be deducted @ 7.5% if the interest payable on 7.75% Savings (Taxable) Bonds,
2018 exceeds during the financial year.
(A) 5,000
(B) 10,000
(C) 20,000
(D) 25,000

B;

Solution: Tax shall be deducted @ 7.5% if the interest payable on 7.75% Savings (Taxable)
Bonds, 2018 exceeds = 10,000 during the financial year.

4. An HUF, not subject to tax audit in the earlier year, paying fees of ₹ 35,000 to a Practising
Company Secretary shall —
(A) Not deduct TDS
(B) Deduct TDS @ 7.5%
(C) Deduct TDS @ 15%
(D) Deduct TDS @ 10.4%.
A;

Solution: Individual or a Hindu undivided family, whose total sales, gross receipts or
turnover from the business or profession carried on by him exceeds ₹ 1 crore and & ₹ 50
lakhs, respectively, during the financial year immediately preceding the financial year in
which such sum by way of fees for professional or technical services is credited or paid, shall
be liable to deduct tax at source. Hence, no tax is to be deducted at source in this case.

5. Any person who is responsible for paying to a resident any income by way of rent u/s 194-I
shall be liable to deduct tax at source where the amount of such income credited or paid or
likely to be credited or paid during the financial year to the account of, or to, the payee,
exceeds .
(A) ₹ 1,80,000
(B) ₹ 10,000
(C) ₹ 2,40,000
(D) ₹ 50,000

C;

Solution: Any person who is responsible for paying to a resident any income by way of rent
shall be liable to deduct tax at source where the amount of such income credited or paid or
likely to be credited or paid during the financial year to the account of, or to, the payee,
exceeds ₹ 2,40,000.

6. The rate of TDS on fees for technical services (not being a professional services) is
(A) 1.5%
(B) 3.75%
(C) 7.5%
(D) 22.5%
A;

Solution: The rate of TDS on fees for technical services (not being a professional services) is
1.5%.

7. Mr. Sam (aged 40 years), a US football match referee, has earned income from football
tournaments in India for A.Y. 2021-22. What are the TDS provisions applicable while making
payment to him?
(A) TDS @ 20.8% as per section 194E
(B) TDS @ 5.2% as per section 194E
(C) TDS under section 195
(D) No tax is deductible at source

C;

Solution: "Match referee" would not fall within the meaning of "sportsmen" to attract the
provisions of Section 115BBA. Therefore, although the payments made to non-resident
"match referee" are "income" which has accrued and arisen in India, the same are not
taxable under the provisions of Section 115BBA. They are subject to the normal rates of tax.
Hence tax on such income will be deducted under Section 195 of the Act.

8. Every seller shall collect tax at source from the buyer at the time of debiting of the amount
payable by the buyer to the account of the buyer; or receipt of such amount from the buyer
in cash or by cheque or draft or any other mode, whichever is earlier, at the rate of
in case of tendu leaves.
(A) 1%
(B) 2.50%
(C) 2%
(D) 3.75%
D;

Solution: Every seller shall collect tax at source from the buyer at the time of debiting of the
amount payable by the buyer to the account of the buyer; or receipt of such amount from
the buyer in cash or by cheque or draft or any other mode, whichever is earlier, at the rate
of 3.75% in case of tendu leaves.

9. In case of individual assessee, amount of advance tax payable in the third installment (i.e.
on or before 15th December) shall be —
(A) 30% of total advance tax payable
(B) 75% of total advance tax — Advance tax paid in earlier installments
(C) 45% of total advance tax — Advance tax paid in earlier installments
(D) 60% of total advance tax — Advance tax paid on earlier installments

B;

Solution: In case of individual assessee, amount of advance tax payable in the third
installment (i.e. on or before 15 th December) shall be 75% of total advance tax — Advance
tax paid in earlier installments.

10. An individual needs to pay ₹ 1,00,000 as advance tax. By 15th of December, how much
amount must be paid by the individual :
(A) ₹ 30,000
(B) ₹ 75,000
(C) ₹ 1,00,000
(D) Nil

B;
Solution: In case of individual assessee, amount of advance tax payable in the third
installment (i.e. on or before 15 th December) shall be 75% of total advance tax — Advance
tax paid in earlier installments. Hence, he should pay ₹ 75,000 upto 15th December.

11. Y Ltd. pays salary of ₹ 2,60,000 (computed) to Mr. X. Mr. X also received interest from
saving bank account ₹ 5,000 & does not have any other income. He paid life insurance
premium of ₹ 15,000 (Policy taken on 01-04-2020 capital sum assured ₹ 2,00,000).
Determine the amount of tax to be deducted by Y Ltd.
(A) ₹ 250
(B) Nil
(C) ₹ 26,500
(D) ₹ 260

B;

Solution: Since the amount of total income do not exceed maximum amount not
chargeable to tax, no tax is to be deducted at source.

Salary Income 2,60,000

Interest on saving bank account 5,000

Gross Total Income 2,65,000

Less: Deduction under Chapter VI-A [Section 80-C — ₹ 15,000] and Section 20,000
80TTA =₹ 5,000

Total Income 2,45,000


12. Mr. A, a salaried individual, pays rent of ₹ 51,000 per month to Mr. B from April 2020.
Which of the statement is true?
(A) No tax is deductible at source since Mr. A is not liable to tax audit u/s 44AB.
(B) Tax is deductible at source every month @ 7.5% on rent paid to Mr. B.
(C) Tax is deductible at source every month @ 3.75% on rent paid to Mr. B.
(D) Tax is deductible at source @ 3.75% on annual rent from the rent paid for March 2020.

D;

Solution: Any person, being an individual or a HUF, other than those individual or HUF
whose total sales, gross receipts or turnover from the business or profession carried on by
him exceed the monetary limits of ₹ 1 crore and ₹ 50 lakhs is responsible for paying to a
resident any income by way of rent, to deduct income tax at the rate of 3.75% only if the
amount of such rent exceeds ₹ 50,000 for a month or part of a month during the previous
year. Hence, Tax is deductible at source @ 3.75% on annual rent from the rent paid for
March 2020.

13. Any person responsible for paying to a non-resident sportsman, who is not a citizen of India
referred under section 115BBA shall be liable to deduct tax at source @ .
(A) 42.744%
(B) 20.8%
(C) 31.2%
(D) 10.4%

B;

Solution: Any person responsible for paying to a non-resident sportsman, who is not a
citizen of India referred under section 115BBA shall be liable to deduct tax at source
@20.8%.
14. On 1-07-2020, Mr. Rajesh started a 9 month recurring deposit of ₹ 1,80,000 per month @
6.25% p.a. with TP Bank. The recurring deposit matures on 31-03-2020. Interest on such
deposit amount is ₹ 42,590. Examine the TDS implications under section 194A.
(A) Tax is required to be deducted by TP Bank on the interest of ₹ 42,590 as it exceeds the
threshold limit of ₹ 40,000
(B) Tax is not required to be deducted by TP Bank on the interest of ₹ 42,590 as it does not
exceeds the threshold limit of ₹ 50,000
(C) Tax Is required to be deducted by TP Bank on the Interest of ₹ 42,590 as it exceeds the
threshold limit of ₹ 5,000
(D) Tax Is required to be deducted by TP Bank on the interest of ₹ 42,590 as it exceeds the
threshold limit of ₹ 10,000

A;

Solution: Tax has to be deducted under section 194A by TP Bank on the interest of ₹ 42,590
falling due on recurring deposit on 31-03-2021 Mr. Rajesh, since —

(a) "recurring deposit" has been included in the definition of "time deposit"; and
(b) such interest exceeds the threshold limit of T 40,000.

15. For what claims under Rule 26C, is employee required to furnish the evidence of such claims
to a person responsible for making payment u/s 192(1) for the purpose of estimating his
income or computing the tax deduction of tax at source?
(A) House Rent Allowance & Leave Travel Concession or Assistance
(B) Deduction of interest under the head Income from house property'
(C) Deduction under Chapter VI-A
(D) All of the above

D;
Solution: Rule 26C has been Inserted in the Income-tax Rules, 1962, with effect from 1st
June, 2016, to require furnishing of evidence of the following claims by an employee to the
person responsible for making payment under section 192(1) In Form No. 12BB for the
purpose of estimating his Income or computing the tax deduction of tax at source :

Nature of Claim Evidence or particulars

1. House Rent Allowance Name, address and PAN of the landlord(s)


where the aggregate rent paid during the
previous year exceeds ₹ 1 lakh.

2. Leave Travel Concession or Assistance Evidence of expenditure

3. Deduction of interest under the head Name, address and PAN of the lender
income from house property"

4. Deduction under Chapter VI-A Evidence of Investment or expenditure.


Transfer Pricing and Other Misc

1. Two enterprises shall be deemed to be associated enterprises if one enterprise holds,


directly or indirectly, shares carrying not less than of the voting power in the
other enterprise.
(A) 51%
(B) 26%
(C) 10%
(D) 20%

B;

Solution: Two enterprises shall be deemed to be associated enterprises if one enterprise


holds, directly or indirectly, shares carrying not less than 26% of the voting power in the
other enterprise.

2. Two enterprises shall be considered as associated enterprises if at any time during the
previous year, following condition is satisfied —
(A) Holding 25% voting power in another enterprise
(B) Advancing of loan which is at least 51% of the book value of the assets of the borrower
enterprise
(C) Providing guarantee for at least 5% of the total borrowings of enterprise
(D) Tapping at least 50% of raw material from the enterprise

B;
Solution: Two enterprises shall be considered as associated enterprises if at any time during
the previous year, a loan advanced by one enterprise to the other enterprise constitutes not
less than 51% of the book value of the total assets of the other enterprise.

3. The transfer pricing provisions shall not apply in a case where the computation of income
has the effect of computed on the basis of entries made in the books of account in
respect of the previous year in which the international transaction was entered into.
(A) Reducing the income chargeable to tax
(B) Increasing the income chargeable to tax
(C) Reducing the loss chargeable to tax
(D) Converting loss into income.

A;

Solution: The transfer pricing provisions shall not apply in a case where the computation of
income has the effect of reducing the income chargeable to tax computed on the basis of
entries made in the books of account in respect of the previous year in which the
international transaction was entered into.

4. A sold a machine to B (associated enterprise) and in turn B sold the same machinery to C
(an independent party) at sale margin of 30% for t 4,00,000 but B has incurred t 4,000 in
sending the machine to C. From the above data, determine arm's length price —
(A) ₹ 2,76,000
(B) ₹ 2,80,000
(C) ₹ 4,00,000
(D) ₹ 1,20,000

A;
Solution: Arms length price will be ₹ 4,00,000 less Gross profit margin 30% i.e. ₹ 1,20,000
less Expenses incurred ₹ 4,000 = ALP = ₹ 2,76,000.

5. Which method is particularly useful where semi-finished goods are transferred between
associated enterprises?
(A) Comparable Uncontrolled Price Method
(B) Resale Price Method
(C) Cost Plus Method
(D) Profit Split Method

C;

Solution: Cost Plus Method is particularly useful where semi-finished goods are transferred
between associated enterprises.

6. In case of Transactional Net Margin Method, where number of value in dataset is less than 6
ALP is computed using—
(A) Range concept
(B) Arithmetic mean of all value in dataset
(C) Any of the price in the data set
(D) Weighted average concept

B;

Solution: In case of Transactional Net Margin Method, where number of value in dataset is
less than 6 ALP is computed using Arithmetic mean of all value in dataset.
7. Rollback provision shall not be provided in respect of an international transaction for a
rollback year, if,-
(A) the determination of arm's length price of the said international transaction for the said
year has been subject matter of an appeal before the Appellate Tribunal and the
Appellate Tribunal has passed an order disposing of such appeal at any time before
signing of the agreement;
(B) the application of rollback provision has the effect of reducing the total income or
increasing the loss, as the case may be, of the applicant as declared
(C) Either (a) or (b)
(D) None of the above

C;

Solution: Rollback provision shall not be provided in respect of an international transaction


for a rollback year, if,-

(i) the determination of arm's length price of the said international transaction for the
said year has been subject matter of an appeal before the Appellate Tribunal and
the Appellate Tribunal has passed an order disposing of such appeal at any time
before signing of the agreement; or
(ii) the application of rollback provision has the effect of reducing the total income or
increasing the loss, as the case may be, of the applicant as declared in the return of
income of the said year.

8. The compliance audit report shall be furnished by the Transfer Pricing Officer within
from the end of the month in which the Annual Compliance Report is received by the
Transfer Pricing Officer.
(A) One month
(B) Six months
(C) Three months
(D) Twelve months

C;

Solution: The compliance audit report shall be furnished by the Transfer Pricing Officer
within six months from the end of the month in which the Annual Compliance Report is
received by the Transfer Pricing Officer.

9. The auditor’s report in respect of transfer pricing shall be in .


(A) Form No. 3CEB
(B) Form No. 3CA
(C) Form No. 3CE
(D) Form No. 3CD

A;

Solution: The auditor’s report in respect of transfer pricing shall be in Form No. 3CEB

10. Which of the following are eligible specified domestic transactions as per Rule 10TH B?
(A) Supply of electricity
(B) Transmission of electricity or wheeling of electricity
(C) Purchase of milk and milk products by a co-operative society from its members.
(D) All of the above

D;

Solution: A specified domestic transaction undertaken by an eligible assessee and which


comprises of:

(i) supply of electricity; or


(ii) transmission of electricity; or

(iii) wheeling of electricity; or

(iv) purchase of milk or milk products by a co-operative society from its members.

11. With whose approval the Board may enter into Advance Pricing Agreement?
(A) Principal Commissioner or Commissioner
(B) Principal Director or the Director
(C) Central Government
(D) Director General of Income-tax (International Taxation)

C;

Solution: The Board, with the approval of the Central Government, may enter into an
advance pricing agreement (APA) with any person, determining the ALP or specifying the
manner in which ALP is to be determined, in relation to an international transaction to be
entered into by that person.

12. Which of the following is not a condition specified under section 94B?
(A) The borrower is an Indian company, or a permanent establishment of a foreign
company in India.
(B) The borrower, incurs any expenditure by way of interest or of similar nature in respect
of any debt.
(C) The interest shall not be deductible in computation of income under the head Profits
and Gains of business or Profession.
(D) The borrower must not be engaged in the business of banking or insurance.

C;
Solution: For the purpose of Section 948 the interest must be deductible in computation of
income under the head Profits and Gains of business or Profession.

13. Every person, being a constituent entity of an international group shall—


(i) if the consolidated group revenue of the international group, of which such person is
a constituent entity, as reflected in the consolidated financial statement of the
international group for the accounting year, exceeds ; and
(ii) the aggregate value of international transactions —
(A) during the accounting year, as per the books of accounts, exceeds or
(B) in respect of purchase, sale, transfer, lease or use of intangible property during
the accounting year, as per the books of accounts, exceeds .
(A) ₹ 500 crore, ₹ 10 crore, ₹ 50 crore
(B) ₹ 500 crore, ₹ 50 crore, ₹ 50 crore
(C) ₹ 500 crore, ₹ 50 crore, ₹ 10 crore
(D) ₹ 50 crore, ₹ 10 crore, ₹ 100 crore

C;

Solution: Every person, being a constituent entity of an international group shall-

(i) if the consolidated group revenue of the international group, of which such person is
a constituent entity, as reflected in the consolidated financial statement of the
international group for the accounting year, exceeds ₹ 500 crore; and
(ii) the aggregate value of international transactions —
(A) during the accounting year, as per the books of accounts, exceeds ₹ 50 crore, or
(B) in respect of purchase, sale, transfer, lease or use of intangible property during
the accounting year, as per the books of accounts, exceeds ₹ 10 crore.
14. "Dispute resolution panel" means a collegium comprising of Principal
Commissioners or Commissioners of Income-tax constituted by the Board for this purpose.
(A) Two
(B) Three
(C) Four
(D) Five

B;

Solution: "Dispute resolution panel" means a collegium comprising of three Principal


Commissioners or Commissioners of Income-tax constituted by the Board for this purpose.

15. If Country A is a notified jurisdictional area (NJA), then, the rate at which interest receivable
from a infrastructure debt fund notified u/s 10(47) is taxable in the hands of Mr. Ram, a
resident of Country A, and the rate at which tax has to be deducted at source on such
income are, respectively,-
(A) 30% and 5%
(B) 5% and 5%
(C) 30% and 30%
(D) 5% and 30%

D;

Solution: As per Section 115A tax on interest receivable from a infrastructure debt fund
notified u/s 10(47) shall be levied @ 5% and as per Section 94A, where any person located
in a notified jurisdictional area is entitled to receive any sum or income or amount on which
tax is deductible under Chapter XVII-B, the tax shall be deducted at the highest of the
following rates,—

(i) at the rate or rates in force ;


(ii) at the rate specified in the relevant provisions of this Act ;

(iii) at the rate of 30%. Hence, tax will be deductible at source @ 30%.

You might also like